Cardio Flashcards

1
Q

What do each of these give rise to?

Truncus arteriosus
Bulbus cordis
Endocardial cushion
Primitive atrium
Primitive ventricle
Primitive pulmonary vein
Left horn of sinus venosus
Right horn of sinus venosus 
Right common cardinal vein and right anterior
cardinal vein 
Right common cardinal vein and right anterior
cardinal vein
A

Truncus arteriosus = Ascending aorta and pulmonary trunk

Bulbus cordis = Smooth parts (outflow tract) of left and right ventricles

Endocardial cushion = Atrial septum, membranous interventricular septum; AV and semilunar valves

Primitive atrium = Trabeculated part of left and right atria

Primitive ventricle = Trabeculated part of left and right ventricles

Primitive pulmonary vein = Smooth part of left atrium

Left horn of sinus venosus = Coronary sinus

Right horn of sinus venosus = Smooth part of right atrium (sinus venarum)

Right common cardinal vein and right anterior
cardinal vein = Superior vena cava (SVC)

How well did you know this?
1
Not at all
2
3
4
5
Perfectly
2
Q

Dextrocardia

A

Occurs during cardiac looping
Primary heart tube loops to establish left-right
polarity; begins in week 4 of gestation.
Defect in left-right dynein (involved in L/R
asymmetry) can lead to dextrocardia, as seen
in Kartagener syndrome (primary ciliary
dyskinesia).

How well did you know this?
1
Not at all
2
3
4
5
Perfectly
3
Q

Steps of chamber formation (atria)

A

pg 268
1. Septum primum grows toward endocardial
cushions, narrowing foramen primum.
2. Foramen secundum forms in septum
primum (foramen primum disappears).
3. Septum secundum develops as foramen
secundum maintains right-to-left shunt.
4. Septum secundum expands and covers most
of the foramen secundum. The residual
foramen is the foramen ovale.
5. Remaining portion of septum primum forms
valve of foramen ovale.
6. (Not shown) Septum secundum and septum
primum fuse to form the atrial septum.
7. (Not shown) Foramen ovale usually closes
soon after birth because of increases LA pressure.

Patent foramen ovale—caused by failure of
septum primum and septum secundum
to fuse after birth; most are left untreated.
Can lead to paradoxical emboli (venous
thromboemboli that enter systemic arterial
circulation), similar to those resulting from
an ASD.

How well did you know this?
1
Not at all
2
3
4
5
Perfectly
4
Q

Steps of ventricular formation (embryo)

A
  1. Muscular interventricular septum forms.
  2. Opening is called interventricular foramen.
  3. Aorticopulmonary septum rotates and fuses with muscular ventricular septum to form membranous interventricular septum, closing
    interventricular foramen.
    *This requires neural crest cells.
  4. Growth of endocardial cushions separates
    atria from ventricles and contributes to both atrial septation and membranous portion of the interventricular septum.

Ventricular septal defect—most common congenital cardiac anomaly, usually occurs in
membranous septum.
-not cyanotic at birth due to L>R shunt

How well did you know this?
1
Not at all
2
3
4
5
Perfectly
5
Q

How are outflow tracts formed and what abnormalities are associated with failure of neural crest cells to migrate?

A

Neural crest and endocardial cell migrations:
forms > truncal and bulbar ridges that spiral
and fuse to form aorticopulmonary septum
forms > ascending aorta and pulmonary trunk.
Conotruncal abnormalities associated with
failure of neural crest cells to migrate:
􀂃 Transposition of great vessels.
􀂃 Tetralogy of Fallot.
􀂃 Persistent truncus arteriosus.

How well did you know this?
1
Not at all
2
3
4
5
Perfectly
6
Q

Describe the 3 major shunts in fetal circulation.
“Blood in umbilical vein has a Po2 of ≈ 30 mm Hg
and is ≈ 80% saturated with O2. Umbilical
arteries have low O2 saturation. “

And then what happens at birth?

Mnemonic: kEEp PDA open

A

3 important shunts:
1. Blood entering fetus through the
umbilical vein is conducted via the ductus
venosus into the IVC, bypassing hepatic
circulation.
2. Most of the highly Oxygenated blood
reaching the heart via the IVC is directed
through the foramen Ovale and pumped
into the aorta to supply the head and body.
3. Deoxygenated blood from the SVC
passes through the RA > RV > main
pulmonary artery > Ductus arteriosus
> Descending aorta; shunt is due to high
fetal pulmonary artery resistance (due
partly to low O2 tension and nonfunctioning lungs).

At birth, infant takes a breath; decrease resistance
in pulmonary vasculature > increased left atrial
pressure vs right atrial pressure; foramen ovale
closes (now called fossa ovalis); increase in O2 (from
respiration) and increase in prostaglandins (from
placental separation) > closure of ductus
arteriosus.
Indomethacin helps close PDA and forms ligamentum
arteriosum (remnant of ductus arteriosus).
Prostaglandins E1 and E2 kEEp PDA open.

How well did you know this?
1
Not at all
2
3
4
5
Perfectly
7
Q

What are the remnants of each of these fetal structures

AllaNtois - urachus  
Ductus arteriosus 
Ductus venosus
Foramen ovale
Notochord
UmbiLical arteries
Umbilical vein
A

AllaNtois - urachus > MediaN umbilical ligament (Urachus is part of allantoic duct between
bladder and umbilicus.)

Ductus arteriosus > Ligamentum arteriosum

Ductus venosus > Ligamentum venosum

Foramen ovale > Fossa ovalis

Notochord > Nucleus pulposus

UmbiLical arteries > MediaL umbilical ligaments

Umbilical vein > Ligamentum teres hepatis (round ligament) (Contained in falciform ligament.)

How well did you know this?
1
Not at all
2
3
4
5
Perfectly
8
Q

pg271

What supples the SA and AV nodes

What does it mean if there is right, left and codominant circulation.

Where does occlusion of the coronary arteries usually occur.

When does coronary blood flow peak.

A

SA and AV nodes are supplied by branches of
the RCA. Infarct may cause nodal dysfunction
(bradycardia or heart block).
(dominance refers to which coronary artery supplies the posterior wall)
Right-dominant circulation (85%) = PDA arises
from RCA.
Left-dominant circulation (8%) = PDA arises
from LCX.
Codominant circulation (7%) = PDA arises
from both LCX and RCA.
Coronary artery occlusion most commonly
occurs in the LAD.
Coronary blood flow peaks in early diastole.

How well did you know this?
1
Not at all
2
3
4
5
Perfectly
9
Q

A 21-year-old man has a mean arterial pressure (MAP) of 89 mm Hg at rest. After running for 40 minutes, his mean arterial pressure has risen only slightly to 99 mm Hg. A decrease in which of the following during exercise most likely accounts for the observed finding?
A. Systolic blood pressure B. Renal blood flow C. Stroke volume D. Systemic vascular resistance E. Right atrial pressure

A

D. Systemic vascular resistance. During exercise, muscle tissue receives up to 85% of blood flow due to vasodilation of muscle vascular beds. In most vascular beds, the SNS causes vasoconstriction via a1 adrenergic receptors. The notable exception is the muscle bed where b2 adrenergic receptors predominant. These receptors when bound by catecholamines cause vasodilation.

How well did you know this?
1
Not at all
2
3
4
5
Perfectly
10
Q

Patient 1
systolic and diastolic BP elevated, PCWP elevated

Patient 2
Systolic and; diastolic BP elevated, PCWP normal

A

Patient 1: mitral stenosis or Eisenmengers

Patient 2: pulmonary embolus (large)

How well did you know this?
1
Not at all
2
3
4
5
Perfectly
11
Q

A man suffers a stroke caused by partial occlusion of his left internal carotid artery. An evaluation of the carotid artery using magnetic resonance imaging (MRI) shows a 75% reduction in its radius. Assuming that blood flow through the left internal carotid artery is 256 mL/min prior to the occlusion, what is blood flow through the artery after the occlusion?

A

A 75% reduction in radius means that the radius is now ¼
of the original value. The resistance has therefore
increased 256 fold (r4). If perfusion pressure remains
constant, flow must then decrease by 256-fold to 1 mL/min.

How well did you know this?
1
Not at all
2
3
4
5
Perfectly
12
Q
A decrease in which of the following would tend to increase lymphatic flow?
A. Hydrostatic capillary pressure 
B. Interstitial osmotic pressure 
C. Plasma osmotic pressure 
D. Capillary filtration constant
A

C plasma osmotic pressure,

if osmotic pressure goes down that means there is less protein

How well did you know this?
1
Not at all
2
3
4
5
Perfectly
13
Q

Normal

EDV = 125
ESV = 50
SV= 75

Patient 1
EDV = 125
ESV = 80
SV= 45

Patient 2
EDV = 125
ESV = 30
SV= 95

Patient 3
EDV = 150
ESV = 50
SV= 100

A

Patient 1 - increased afterload

Patient 2: increase contractility

Patient 3: increased preload

How well did you know this?
1
Not at all
2
3
4
5
Perfectly
14
Q

A 40-year-old male is hospitalized with progressive exertional dyspnea, lower extremity edema and cough. She also describes frequent nocturnal episodes of breathlessness and recent hoarseness. Auscultation reveals a mid-diastolic rumble best heard at the cardiac apex. This patient’s hoarseness is most likely caused by:
A. Laryngeal edema B. Impaired arterial supply C. Nerve impingement D. Epithelial sloughing E. Vocal cord polyps

A

C. The patient has mitral stenosis resulting in LA enlargement and pressure on the L recurrent laryngeal nerve (see arrow).

This is Ortner’s syndrome

How well did you know this?
1
Not at all
2
3
4
5
Perfectly
15
Q
  1. An 80 year old male with syncope on exertion and weak carotid pulses
  2. A 26 year old female IV drug user with fever, chills and prominent head bobbing
  3. A 54 year old male with blood tinged sputum and a history of rheumatic heart disease
  4. A 39 year old women with nocturnal dyspnea and a systolic blowing murmur
    (aortic/mitral, stenosis/regurg)?
A
  1. An 80 year old male with syncope on exertion and weak carotid pulses - aortic stenosis
  2. A 26 year old female IV drug user with fever, chills and prominent head bobbing - endocarditis destroys the valve, head bobbing suggests aortic regurg
  3. A 54 year old male with blood tinged sputum and a history of rheumatic heart disease - before RHD causes mitral/tricuspid regurg it classically causes mitral stenosis), blood is backing up into the lungs.
  4. A 39 year old women with nocturnal dyspnea and a systolic blowing murmur - mitral insufficiency (blowing mumur and nocturnal dyspenea)
How well did you know this?
1
Not at all
2
3
4
5
Perfectly
16
Q

Alpha -1 receptor functions and alpha 2 receptor functions

A

alpha 1

  1. **vasoconstriction (increased peripheral vascular resistance and increased BP) - no effect on heart
  2. mydriasis (ciliary body contracts, increases intraocular pressure because block outflow tract)
  3. Bladder sphincter constriction

alpha 2:

  1. **inhibit NE release (decrease sympathetic activity, decreasing blood pressure)
  2. inhibit insulin release from beta cells
  3. decrease aqueous humor production
How well did you know this?
1
Not at all
2
3
4
5
Perfectly
17
Q

Beta 1 receptor

Beta 2 receptor

Beta 3 receptor

functions

A

Beta 1: (heart)

  • tachycardia
  • increased contractility
  • increased renin (increase BP)

Beta 2:

  • vasodilation, decrease peripheral vascular resistance (mostly skeletal)
  • bronchodilator
  • 🌸uterine relaxation
  • increase glucose (glycogenolysis, gluconeogenesis)
  • 🍌increase potassium uptake (can treat hyperkalemia)

Beta 3: lipolysis

How well did you know this?
1
Not at all
2
3
4
5
Perfectly
18
Q

Familial Hypercholesterolemia

A

Autosomal dominant disorder
-severe elevations in total cholesterol and LDL-C

  1. (most common) nonsense mutation in the LDL receptor
  2. APO-B100, inhibits binding of LDL to LDL-R
  3. PCSK9
    - gain of function
    - enhances LDL-R degradation

Homozygous FH - severely elevated cholesterol levels (total cholesterol and LDLc levels >600 mg/dL); TAG levels are within normal limits

Heterozygous FH - elevated LDLc levels commonly greater than 250 mg/dL

How well did you know this?
1
Not at all
2
3
4
5
Perfectly
19
Q

Hyperchylomicronemia (type 1)

A

-eruptive xanthoma (as opposed to tendonous xanthoma/LDL-R defect) or (tuberous xanthomas - ApoE defect -don’t take VLDL or IDL) - it looks different, pimples become yellow, not itchy)

  • very high serum triglycerides
  • cholesterol is usually normal
  • deficiency in lipoprotein lipase OR in ApoCII
  • associated with pancreatitis
  • no increased risk of coronary artery disease
How well did you know this?
1
Not at all
2
3
4
5
Perfectly
20
Q

Describe the fetal circulation

A

Blood in umbilical vein has a Po2 of ≈ 30 mmHg
and is ≈ 80% saturated with O2. Umbilical
arteries have low O2 saturation.
3 important shunts:
1. Blood entering fetus through the
umbilical vein is conducted via the ductus
venosus into the IVC, bypassing hepatic circulation.
2. Most of the highly Oxygenated blood
reaching the heart via the IVC is directed through the foramen Ovale and pumped into the aorta to supply the head and body.
3. Deoxygenated blood from the SVC
passes through the RA >RV > main
pulmonary artery >Ductus arteriosus
> Descending aorta; shunt is due to high fetal pulmonary artery resistance (due partly to low O2 tension).
At birth, infant takes a breath; decrease resistance in pulmonary vasculature >increase left atrial
pressure vs right atrial pressure; foramen ovale
closes (now called fossa ovalis); increase in O2 (from respiration) and increase in prostaglandins (from
placental separation) > closure of ductus arteriosus.
Indomethacin helps close PDA > ligamentum arteriosum (remnant of ductus arteriosus). Prostaglandins E1 and E2 kEEp PDA open.

How well did you know this?
1
Not at all
2
3
4
5
Perfectly
21
Q

Ostium Secundum Type
Ostium Primum Type
Patent foramen ovale

A

🌟Ostium Secundum Type (90%) is caused
by abnormal growth (not enough) of the
septum secundum.

• Ostium Primum Type (5%) is where the septum primum doesn’t fuse with the endocardial cushion. Seen in Down
syndrome and associated with AV valve
defects.

• Patent foramen ovale is common (25% of adults) and is caused by failure of septum primum and septum secundum to fuse after birth (there is no true hole in the septum at rest). Not considered a true ASD and treatment (closure) is controversial.

How well did you know this?
1
Not at all
2
3
4
5
Perfectly
22
Q

Wolff-Parkinson-White syndrome

A

Most common type of ventricular pre-
excitation syndrome. Abnormal fast accessory conduction pathway from atria to ventricle (bundle of Kent) bypasses the rate-slowing
AV node >ventricles begin to partially
depolarize earlier > characteristic delta wave with widened QRS complex and shortened PR interval on ECG. May result in reentry circuit
>supraventricular tachycardia.

How well did you know this?
1
Not at all
2
3
4
5
Perfectly
23
Q

Right-to-left shunt

A

Early cyanosis—“blue babies.” Often diagnosed
prenatally or become evident immediately
after birth. Usually require urgent surgical treatment and/or maintenance of a PDA.

The 5 Ts:

  1. Truncus arteriosus (1 vessel)
  2. Transposition (2 switched vessels)
  3. Tricuspid atresia (3 = Tr i)
  4. Tetralogy of Fallot (4 = Te t r a)
  5. TAPVR (5 letters in the name)
How well did you know this?
1
Not at all
2
3
4
5
Perfectly
24
Q

Persistent truncus arteriosus

A

Truncus arteriosus fails to divide into
pulmonary trunk and aorta due to lack of
aorticopulmonary septum formation; most patients have accompanying VSD.

How well did you know this?
1
Not at all
2
3
4
5
Perfectly
25
Q

Transposition of the great vessels

A

Aorta leaves RV (anterior) and pulmonary trunk
leaves LV (posterior) > separation of systemic
and pulmonary circulations. Not compatible with life unless a shunt is present to allow mixing of blood (eg, VSD, PDA, or patent
foramen ovale).
Due to failure of the aorticopulmonary septum to spiral.
Without surgical intervention, most infants die within the irst few months of life.

How well did you know this?
1
Not at all
2
3
4
5
Perfectly
26
Q

Tricuspid atresia

A

Absence of tricuspid valve and hypoplastic RV;
requires both ASD and VSD for viability.

Tricuspid atresia is a form of congenital heart disease whereby there is a complete absence of the tricuspid valve. Therefore, there is an absence of right atrioventricular connection. This leads to a hypoplastic (undersized) or absent right ventricle. This defect is contracted during prenatal development, when the heart does not finish developing. It causes the heart to be unable to properly oxygenate the rest of the blood in the body. Because of this, the body does not have enough oxygen to live, so other defects must occur to maintain blood flow. ⭐️Because of the lack of an A-V connection, an atrial septal defect (ASD) must be present to fill the left ventricle with blood. Also, since there is a lack of a right ventricle there must be a way to pump blood into the pulmonary arteries, and this is accomplished by a ventricular septal defect (VSD). The causes of Tricupsid atresia are unknown.[1]

How well did you know this?
1
Not at all
2
3
4
5
Perfectly
27
Q

Tetraology of Fallot

MnemonicL PROVe

A

Caused by anterosuperior displacement of the
infundibular septum. Most common cause of early childhood cyanosis

  1. Pulmonary infundibular stenosis (most important determinant for prognosis)
  2. Right ventricular hypertrophy (RVH)—boot-shaped heart on CXR
  3. Overriding aorta
  4. VSD

Pulmonary stenosis forces right-to-left flow across VSD > RVH, “tet spells” (often caused by crying, fever, and exercise due to exacerbation of RV outlow obstruction).

PROVe. Squatting: increases SVR, decreases right-to-left shunt, improves cyanosis.
Treatment: early surgical correction.

How well did you know this?
1
Not at all
2
3
4
5
Perfectly
28
Q

Total anomalous pulmonary venous return

A

Pulmonary veins drain into right heart circulation (SVC, coronary sinus, etc);
associated with ASD and sometimes PDA to allow for right-to-left shunting to maintain CO

How well did you know this?
1
Not at all
2
3
4
5
Perfectly
29
Q

Ebstein anomaly

A

Characterized by displacement of tricuspid
valve lealets downward into RV, artiicially
“atrializing” the ventricle. Associated with tricuspid regurgitation and right heart failure. Can be
caused by lithium exposure in utero.

How well did you know this?
1
Not at all
2
3
4
5
Perfectly
30
Q

Left-to-right shunts

eaRLy
LateR

A

Acyanotic at presentation; cyanosis may occur
years later.
Frequency: VSD > ASD > PDA.

Right-to-Left shunts: eaRLy cyanosis. Left-to-Right shunts: “LateR” cyanosis.

How well did you know this?
1
Not at all
2
3
4
5
Perfectly
31
Q

Ventricular septal defect

A

Most common congenital cardiac defect.
Asymptomatic at birth, may manifest week later or remain asymptomatic throughout life.
Most self resolve; larger lesions may lead to LV overload and HF.

O2 saturation increase in RV and pulmonary artery

How well did you know this?
1
Not at all
2
3
4
5
Perfectly
32
Q

Atrial septal defect

A

Defect in interatrial septum; loud S1; wide,
fixed split S2. Ostium secundum defects
most common and usually occur as isolated findings; ostium primum defects rarer yet usually occur with other cardiac anomalies.
Symptoms range from none to HF. Distinct from patent foramen ovale in that septa are missing tissue rather than unfused.

O2 saturation increases in RA, RV, and pulmonary artery. May lead to paradoxical emboli
(systemic VENOUS emboli use ASD to bypass lungs and become systemic arterial emboli).

How well did you know this?
1
Not at all
2
3
4
5
Perfectly
33
Q

Patent Ductus Arteriosus

ENDomethacin
PGE

A

In fetal period, shunt is right to left (normal).
In neonatal period, decrease pulmonary vascular
resistance > shunt becomes left to right > progressive RVH and/or LVH and HF.
Associated with a continuous, “machine-like murmur. Patency is maintained by PGE synthesis and low O2 tension. Uncorrected PDA can eventually result in late cyanosis
in the lower extremities (differential cyanosis).

Endomethacin” (indomethacin) ends patency of PDA; PGE keeps ductus Going (may be necessary to sustain life in conditions such as transposition of the great vessels). PDA is normal in utero and normally closes only
after birth.

How well did you know this?
1
Not at all
2
3
4
5
Perfectly
34
Q

Eisenmenger syndrome

A

Uncorrected left-to-right shunt (VSD, ASD, PDA) > increase pulmonary blood flow >pathologic
remodeling of vasculature >pulmonary
arterial hypertension. RVH occurs to
compensate > shunt becomes right to
left. Causes late cyanosis, clubbing D , and polycythemia. Age of onset varies.

How well did you know this?
1
Not at all
2
3
4
5
Perfectly
35
Q

Coarctation of the aorta

A

Infantile:
Preductal coarctation - (Turner Syndrome, FA p585) - constriction is proximal to the DA. Before birth, blood flows through the DA to the descending aorta for distribution to the lower body. Collaterals DO NOT NEED develop in utero. For this reason, preductal coarctation can be life-threating in neonate. Cyanosis from birth immediately symptomatic

Adult: there is no PDA, the coarctation is after the major branches of the aorta so intercostals have to form. Blood can’t flow well so instead more are forced out into the major branches leading to hypertension in upper extremities and hypotension in lower extremities. Intercostals will try to bypass the coarctation and lead to rib notching. The renals are in the hypotense region so they think there is hypotension and activate the RAAS.
______________________________

FA: Aortic narrowing near insertion of ductus arteriosus (“juxtaductal”). Associated with bicuspid aortic valve, other heart defects, and Turner syndrome.
Hypertension in upper extremities and weak, delayed pulse in lower
extremities (brachial-femoral delay). With age, intercostal arteries enlarge due to collateralcirculation; arteries erode ribs > notched
appearance on CXR.
Complications include HF,risk of cerebral hemorrhage (berry
aneurysms), aortic rupture, and possible endocarditis.

How well did you know this?
1
Not at all
2
3
4
5
Perfectly
36
Q

Aortic arch derivatives

A

Aortic arch derivatives

1st: Part of maxillary artery (branch of external
carotid).
1st arch is maximal

2nd: Stapedial artery and hyoid artery.
Second = Stapedial

3rd: Common Carotid artery and proximal part of internal Carotid artery
C is 3rd letter of alphabet

4th: On left, aortic arch; on right, proximal part of right subclavian artery.
4th arch (4 limbs) -systemic 

🌈6th: Proximal part of pulmonary arteries and (on left only) ductus arteriosus
*6th arch = pulmonary and pulmonary -to systemic shunt (ductus arteriosus

*Right recurrent laryngeal nerve loops around right subclavian artery
4th
Left recurrent laryngeal nerve loops around aortic arch distal to ductus arteriosus

How well did you know this?
1
Not at all
2
3
4
5
Perfectly
37
Q

Measurements of blood oxygen are taken in a fetus. The highest value is most likely recorded in which of the following vessels?

A. Ductus arteriosus B. Superior vena cava C. Inferior vena cava D. Pulmonary trunk E. Ascending aorta F. Descending aorta G. Umbilical artery

A

C. inferior vena cava

How well did you know this?
1
Not at all
2
3
4
5
Perfectly
38
Q

A 5-year-old male with a bounding pulse has a thrill best palpated over the upper left sternal edge. A continuous murmur is heard over the area on cardiac auscultation and caused by the lack of closure of fetal bypass channel. This channel is a derivative of which of the following embryologic structures?

A. Sinus venosus B. Bulbus cordis C. Primitive atria D. Fourth aortic arch E. Sixth aortic arch

A

E. sixth aortic arch - ductus arteriosus

How well did you know this?
1
Not at all
2
3
4
5
Perfectly
39
Q

Rapid fire

  1. What defines dominance of circulation
  2. What is supplying the SA and AV node
  3. Where does coronary artery occlusion most commonly occur
  4. What are some consequences of left atrial enlargement
A

1) Right-dominant circulation=85%; defined by which artery supplies the PDA - supplies posterior 1/3 of interventricular septum, AV node and posterior wall of ventricles
2) SA and AV node usually supplied by RCA
3) Coronary artery occlusion most commonly occurs in the LAD
4) Left atrium enlargement can cause dysphagia (compression of the esophagus) or hoarseness (compression of the left recurrent laryngeal nerve, a branch of the vagus).

How well did you know this?
1
Not at all
2
3
4
5
Perfectly
40
Q

Myocardial/Ventricular Action Potential

Phase 0-4

In contrast to skeletal muscle:
􀂃 Cardiac muscle action potential has a plateau, which is due to Ca2+ influx and K+ efflux.

􀂃 Cardiac muscle contraction requires Ca2+ influx from ECF to induce Ca2+ release from sarcoplasmic reticulum (Ca2+-induced Ca2+ release)
.
􀂃 Cardiac myocytes are electrically coupled to each other by gap junctions.

A

Phase 0 = Rapid upstroke – Voltage-gated (VG) Na+ channel open.

Phase 1 = Initial repolarization – inactivation of VG Na+ channels. VG K+ channel begins to open.

Phase 2 = Plateau – Ca2+ influx through VG Ca2+ channels balances K+ efflux. Ca2+ influx triggers Ca2+ release from sarcoplasmic reticulum and myocyte contraction

Phase 3 = Rapid repolarization – massive K+ efflux due to opening of VG slow K+ channels and closure of VG Ca2+ channels.

Phase 4 = Resting potential – high K+ permeability through K+ channels

How well did you know this?
1
Not at all
2
3
4
5
Perfectly
41
Q

Equations for:

CO

SV

EF

Wall tension

MAP

Resistance of arterioles

Fick’s principle

Pulse pressure

A
  1. CO = SV x HR (ml or L/min)
  2. SV = EDV-ESV
  3. EF = SV/EDV normal is >55%
  4. Wall tension = (Pressure x radius)/(2 x thickness)
  5. MAP = CO x TPR (aka SVR), MAP = 2/3 diastolic + 1/3 systolic
  6. Resistance - viscocity x length / radius ^4
  7. Fick’s principle: CO = (rate of O2 consumption)/
    (arterial O2 content − venous O2 content
  8. Pulse pressure = systolic pressure – diastolic pressure
    *Pulse pressure is proportional to SV, inversely
    proportional to * arterial compliance.
How well did you know this?
1
Not at all
2
3
4
5
Perfectly
42
Q

What are the steps to catecholamine induced increase in inotropy

A
  1. Phosphorylation of Ca2+ channels causes the Ca2+ channels to remain open ⏱longer. - increase contractility with increase intracellular calcium
  2. Phosphorylation of proteins in the SR enhances release of Ca2+. - increase contractility with increase intracellular calcium
  3. Phosphorylation of myosin increases myosin ATPase which increases the speed of cross-bridge cycling.
  4. Phosphorylation of Ca2+ pumps in the SR increases the speed of calcium re-uptake, and relaxation.
How well did you know this?
1
Not at all
2
3
4
5
Perfectly
43
Q

What is the NCX? Sodium calcium exchanger

How is this involved in the effects of digitalis (digoxin)

A

The sodium-calcium exchanger (often denoted NCX) removes calcium from cells, using the electrochemical gradient of sodium (Na+). Calcium out, sodium in. When the exchanger works poorly, calcium accumulates in the myocyte leading to enhanced contractility.

First of all digitalis increases inotropy and SV. It does this not by directly affecting the NCX.
Digitalis competes with K+ for binding to the Na+/K+ ATPase. (Potassium in, sodium out) Therefore hypokalemia increases risk of drug toxicity.

Therefore sodium builds up intracellularly and the NCX can’t bring sodium in and calcium out so calcium builds intracellularly

How well did you know this?
1
Not at all
2
3
4
5
Perfectly
44
Q

myoCARDial oxygen consumption

A

increased by Contractility, Afterload, heart Rate, and ventricular Diameter

How well did you know this?
1
Not at all
2
3
4
5
Perfectly
45
Q

Why do you think an increase in preload (reflected by SV) increases MVO2 less than an increase in afterload (reflected by MAP)?

A

D. Radius increases by the 3rd root of volume assuming the LV is a sphere

How well did you know this?
1
Not at all
2
3
4
5
Perfectly
46
Q

Eccentric and concentric hypertrophy

A

Cardiomyocytes grow in both length and width by addition of sarcomeres. Physiological stimulation (such as pregnancy or endurance training) can induce eccentric hypertrophy. Here sarcomeres are added to increase length but cellular width also increases somewhat.
Physiologic concentric hypertrophy is seen in isometric exercise training, such as weight-lifting (increase in sarcomere width only).

Pathologic eccentric hypertrophy features increases in sarcomere but cells shrink in width (e.g. valvular regurgitation).
Pathologic concentric hypertrophy arises in chronic hypertension or aortic stenosis.

How well did you know this?
1
Not at all
2
3
4
5
Perfectly
47
Q

Using Laplace’s equation which hypertrophy strategy or strategies would be most effective in reducing O2 consumption?
A. Concentric because wall tension would increase
B. Concentric because wall tension would decrease
C. Eccentric because wall tension would increase
D. Eccentric because wall tension would decrease E. Both strategies because wall tension would increase with either F. Both strategies because wall tension would decrease with either

A

B> concentric because wall tension would decrease

How well did you know this?
1
Not at all
2
3
4
5
Perfectly
48
Q
A 21-year-old man has a mean arterial pressure (MAP) of 89 mm Hg at rest. After running for 40 minutes, his mean arterial pressure has risen only slightly to 99 mm Hg.  A decrease in which of the following during exercise most likely accounts for the observed finding?
A. Systolic blood pressure 
B. Renal blood flow 
C. Stroke volume 
D. Systemic vascular resistance 
E. Right atrial pressure
A

D. Systemic vascular resistance. During exercise, muscle tissue receives up to 85% of blood flow due to vasodilation of muscle vascular beds. In most vascular beds, the SNS causes vasoconstriction via a1 adrenergic receptors. The notable exception is the muscle bed where b2 adrenergic receptors predominant. These receptors when bound by catecholamines cause vasodilation.

How well did you know this?
1
Not at all
2
3
4
5
Perfectly
49
Q

What increases pulse pressure
What decreases pulse pressure
FA. 272

A

Increased pulse pressure in…
-hyperthyroidism,
-aortic regurgitation (drops diastolic because blood has another route),
-aortic stiffening (arteriosclerosis in elderly - can’t absorb pressure
-exercise (transient) increases BP and decreases SVR)
-obstructive sleep
apnea (increases sympathetic tone)

Decreased pulse pressure in

  • aortic stenosis,
  • cardiogenic shock,
  • cardiac tamponade
  • advanced heart failure (HF).
How well did you know this?
1
Not at all
2
3
4
5
Perfectly
50
Q

hydrostatic pressure vs osmotic pressure

and what does net filtration pressure mean?

A

Hydrostatic pressure - is about fluid movement OUT

Osmotic pressure - is about solute concentration drawing fluid IN

Net filtration pressure, the higher it is, the higher the movement OUT of capillaries

How well did you know this?
1
Not at all
2
3
4
5
Perfectly
51
Q

What are causes of edema?

A

1) Increased capillary pressure (increased Pc, heart failure)
2) Decreased plasma proteins (decreased πc, nephrogenic syndrome, liver failure)
3) Increased capillary permeability (increased Kf, toxins, infections, burns)
4) Increased interstitial fluid colloid osmotic pressure (increased π lymphatic blockage)

How well did you know this?
1
Not at all
2
3
4
5
Perfectly
52
Q

*What are the heart sounds

A

S1- mitral and tricuspid valve closure. Loudest at mitral area

S2- aortic and pulmonary valve closure. Loudest at left upper sternal border

S3 (abnormal)- in early diastole during rapid ventricular filling phase. Associated with increased filling pressures (e.g. CHF) and more common in dilated ventricles. Can be normal in children and pregnant women.

S4 (abnormal)(“atrial kick”) – in late diastole. High atrial pressure. Associated with ventricular hypertrophy. Left atrium must push against stiff LV wall. - “stiff noncompliant ventricle during left atrial contraction).

How well did you know this?
1
Not at all
2
3
4
5
Perfectly
53
Q

Heart sound splitting

A

Normal splitting – Inspiration > drop in intrathoracic pressure > increased venous return to the RV > increased RV stroke volume and ejection time > delayed closure of pulmonic valve (P2).
Also decreased pulmonary impedance and therefore increased capacity of the pulmonary circulation

Wide splitting –Delay in RV emptying causes delayed P2 (regardless of breath but especially on inspiration). An exaggeration of normal splitting.
-Right bundle branch block, pulmonary stenosis

Fixed splitting – Left to right shunt, increased RA and RV volume, regardless of breath. P2 will always be greatly delayed.
*Atrial septal defect

Paradoxical splitting – Delay in LV emptying. Normal order of valve closure is reversed so that P2 sound occurs before delayed A2 sound.
-left bundle branch
-aortic stenosis
Therefore on inspiration,
P2 closes later and moves closer to A2, thereby
“paradoxically” eliminating the split (usually
heard in expiration).

How well did you know this?
1
Not at all
2
3
4
5
Perfectly
54
Q

Heart murmurs

MR/TR
AS
VSD
Mitral valve prolapse

A

[Systolic]
Holosystolic, high-pitched “blowing murmur.”
Mitral—loudest at apex and radiates toward axilla. MR is often due to ischemic heart disease (post-MI), mitral valve prolapse, LV dilatation.
Acute: endocarditis, papillary muscle rupture, trauma, chordal rupture)
Chronic: myxomatous, RF, healed endocarditis, radiation)

Tricuspid—loudest at tricuspid area. TR commonly caused by RV dilatation.
Rheumatic fever and infective endocarditis can cause either MR or TR.

Aortic stenosis (AS):
Crescendo-decrescendo systolic ejection murmur (ejection click may be present).
LV >> aortic pressure during systole. Loudest at heart base; radiates to carotids.
“🌌Pulsus parvus et tardus”—pulses are weak with a delayed peak. Can lead to Syncope, Angina, and Dyspnea on exertion (SAD). Most commonly due to agerelated calcification in older patients (> 60 years old) or in younger patients with early-onset calcification of bicuspid aortic valve.

VSD Loudest at tricuspid area with a L to R shunt. Harsh holosystolic murmur in physical exam.

Mitral Valve prolapse (MVP) Late systolic crescendo murmur with midsystolic click (MC; due to sudden tensing of chordae tendineae). **Most frequent valvular lesion. Best heard over apex. Loudest
just before S2. Usually benign. Can predispose to infective endocarditis. Can be
caused by myxomatous degeneration (1° or 2° to connective tissue disease such as
Marfan or Ehlers-Danlos syndrome), rheumatic fever, chordae rupture.
__________________________

How well did you know this?
1
Not at all
2
3
4
5
Perfectly
55
Q
A 40-year-old male is hospitalized with progressive exertional dyspnea, lower extremity edema and cough. She also describes frequent nocturnal episodes of breathlessness and recent hoarseness.  Auscultation reveals a mid-diastolic rumble best heard at the cardiac apex. This patient's hoarseness is most likely caused by:
A. Laryngeal edema 
B. Impaired arterial supply 
C. Nerve impingement 
D. Epithelial sloughing 
E. Vocal cord polyps
A

C. The patient has mitral stenosis resulting in LA enlargement and pressure on the L recurrent laryngeal nerve (see arrow). This is Ortner’s syndrome

How well did you know this?
1
Not at all
2
3
4
5
Perfectly
56
Q
  1. An 80 year old male with syncope on exertion and weak carotid pulses
  2. A 26 year old female IV drug user with fever, chills and prominent head bobbing
  3. A 54 year old male with blood tinged sputum and a history of rheumatic heart disease
  4. A 39 year old women with nocturnal dyspnea and a systolic blowing murmur

AS, AR, MS, MR - heart

A
  1. Aortic stenosis
  2. Aortic regurg
  3. Mitral stenosis
  4. Mitral regurg
How well did you know this?
1
Not at all
2
3
4
5
Perfectly
57
Q

Describe pacemaker action potential

A

Phase 0= Upstroke- VG Ca2+ channels open. VG Na+ channels are inactivated at less negative resting voltage.

Phase 1 and 2 = absent

Phase 3= inactivation of the Ca2+ channels and increase activation of the K+ channels resulting in K+ efflux

Phase 4= slow spontaneous diastolic depolarization due to If (“funny current”). If channels
responsible for a slow, mixed Na+/K+ inward current; different from INa in phase 0 of ventricular
action potential. Accounts for automaticity of SA and AV nodes. The slope of phase 4 in the SA
node determines HR. ACh/adenosine decrease the rate of diastolic depolarization and decrease HR, while
catecholamines increase depolarization and increase HR. Sympathetic stimulation increase the chance that If channels are open and thus increase HR.

How well did you know this?
1
Not at all
2
3
4
5
Perfectly
58
Q

Name the AV blocks

A

1st degree – PR interval is prolonged (>200 msec). Benign and asymptomatic. No treatment required.

2nd degree
-Mobitz type I (Wenckebach)- Progressive lengthing of the PR interval until a beat is “dropped”. Usually asymptomatic

-Mobitz Type II – Dropped beats that are not preceded by a change in length of the PR interval (as in type I). May progress to 3rd- degree block. Often treated with pacemaker. (one beat drop 2)

3rd degree (complete) –Both P waves and QRS complexes are present, although the P waves bear no relation to the QRS complexes. The atrial rate is faster than ventricular rate. Usually treated with pacemaker. Lyme disease and MIs are causes.

How well did you know this?
1
Not at all
2
3
4
5
Perfectly
59
Q

Barroreceptor reflex

A

Baroreceptor Reflex
Aortic arch transmits via CN X. Carotid sinus transmits via CN IX.

High BP activates baroreceptor activity, which the solitary nucleus integrates and then activates the PSNS (and inhibits the SNS) to slow heart rate and reduce BP

Low BP reduces baroreceptor activity, which the solitary nucleus integrates and activates the SNS (and inhibits the PSNS) to increase HR and increase BP

How well did you know this?
1
Not at all
2
3
4
5
Perfectly
60
Q

A 20-year-old female presents to the physician following a near syncopal episode. His blood pressure is 90/60 mmHg and his heart rate is 180/min and regular. Gentle neck massage just below the angle of the right mandible produces an immediate reduction in his heart rate to 72 bpm and his BP increased to 110/70. This maneuver (carotid massage) improved the patient’s symptoms by:

A

This is a patient with paroxysmal supraventricular tachycardia (PSVT), the most common paroxysmal tachycardia. PSVT typically results from a re-entrant impulse traveling through slowly and rapidly conducting segments of the AV node. Simple treatments include maneuvers that increase cardiac PSNS tone, such as carotid sinus massage and the Valsalva maneuver. The PSNS primarily functions to slow the heart rate by slowing conduction through the AV node, which is specifically beneficial for this arrhythmia because it abolishes the re-entrant circuit and the arrhythmia.

How well did you know this?
1
Not at all
2
3
4
5
Perfectly
61
Q

What are noncardiac effects of alpha1 stimulation

A
  1. 💦Bladder outlet obstruction/smooth muscle constriction aka urinary retention.
  2. Mydriasis - ciliary body contracts
  3. Increased intraocular pressure - reduces reuptake of aqueous humor, acute angle closure glaucoma.
How well did you know this?
1
Not at all
2
3
4
5
Perfectly
62
Q

Besides inhibiting NE and acetylcholine release from presynaptics, what noncardiac function do alpha 2 receptors play?

A

They affect pancreatic beta cells and also inhibition of insulin release

They also decrease production of aqueous humor.

Alpha 2, always think blocking release.

How well did you know this?
1
Not at all
2
3
4
5
Perfectly
63
Q

What factors increase contractility?

What factors decrease

A

Contractility (and SV) increase with:
-Catecholamine stimulation via β1 receptor:
ƒ -Ca2+ channels phosphorylated >increasedCa2+
entry >increasedCa2+-induced Ca2+ release and increasedCa2+ storage in sarcoplasmic reticulum ƒ

-Phospholamban phosphorylation >active Ca2+ ATPase >increased Ca2+ storage in sarcoplasmic reticulum - a protein that negatively regulates SERCA, increasing the uptake and storage of calcium in SR

ƒ-increased intracellular Ca2+ ƒ
-decreased extracellular Na+ (decreased activity of Na+/Ca2+
exchanger) ƒ
-Digitalis (blocks Na+/K+ pump
> increased intracellular Na+ > decreased Na+/Ca2+
exchanger activity > increased intracellular Ca2+)

Contractility and SV decrease with:
-β1-blockade (decreased cAMP)
ƒ -HF with systolic dysfunction
ƒ -Acidosis
ƒ -Hypoxia/hypercapnia (decrease Po2/ increase Pco2)
ƒ- Non-dihydropyridine Ca2+ channel blockers

How well did you know this?
1
Not at all
2
3
4
5
Perfectly
64
Q

What are the function of dopamine receptors in the cardiac system?

A
  • Natriuresis

- Vascular smooth muscle relaxation (renal splanchnic)

How well did you know this?
1
Not at all
2
3
4
5
Perfectly
65
Q

What are properties of normal endothelium?

A

Endothelium dependent vasoregulation

  • Dilation: NO, Prostacyclin
  • Endothelin: strongest vasoconstrictor substances

Antihypertrophic properties
-inhibition of vascular smooth muscle cells proliferation and migration

Modulation of immune response
-resist WBC adhesion and attachment

Anticoagulant, antithrombotic, profibrinolytic function

  • heparan sulfate
  • thrombomodulin
How well did you know this?
1
Not at all
2
3
4
5
Perfectly
66
Q

Steps of atherosclerosis

A
  1. damage to endothelium (large,medium sized muscular arteries especially at bifurcations) - disturbed shear features low flow, flow reversal and an altered EC phenotype and behavior
    - high EC turnover, poor alignment, inflammatory genes, high permeability, oxidative stress (ROS), prothrombotic - prostacyclin and NO decreased
  2. LDL oxidation
  3. Scavenger receptor - macrophages take up oxidized LDL becoming foam cell
  4. Oxidized LDL and damage signals expression of adhesion molecules, monocyte attachment, transmigration, monocyte activation into more macrophages.
  5. Cell necrosis leaves a lipid pool
  6. Macrophage cytokines stimulate smooth muscle cells migration and replication, matrix synthesis eventually forming fibrous cap.
  7. Active macrophages eventually weaken the plaque cap, exposure of matrix to platelets and mural thrombus formation.
How well did you know this?
1
Not at all
2
3
4
5
Perfectly
67
Q

Turner syndrome

A

Female (45, XO)
Short stature (if untreated; preventable with growth hormone therapy), ovarian dysgenesis (streak ovary), shield chest, bicuspid aortic valve, coarctation (femoral < brachial pulse),
lymphatic defects (result in webbed neck or cystic hygroma; lymphedema in feet, hands) horseshoe kidney. Most common cause of 1° amenorrhea. No Barr body.
Menopause before menarche. decrease estrogen leads to increase LH, FSH. Sometimes due to mitotic error - mosaicism (eg,
45,XO/46,XX). Pregnancy is possible in some cases (IVF, exogenous estradiol-17β and progesterone).

Congenital defect: Bicuspid aortic valve, coarctation of aorta, dissection of aorta

Most girls and women with Turner syndrome have normal intelligence. Developmental delays, nonverbal learning disabilities, and behavioral problems are possible, although these characteristics vary among affected individuals.

How well did you know this?
1
Not at all
2
3
4
5
Perfectly
68
Q

A man starts taking metoprolol, how would each of the following parameters change
Cardiac myocyte [camp]
JGA cell [camp]
Vascular smooth muscle [cAMP]

A

Cardiac myocyte [camp] - decreased

JGA cell [camp] decreased

Vascular smooth muscle [cAMP] no change

How well did you know this?
1
Not at all
2
3
4
5
Perfectly
69
Q

A 48 yr old woman comes to her physicians office with a 1 month history of dyspena on exertion that resolves with rest. Her other medical problems include hypertension and hypercholesterolemia. An exercise stress test reveals ST segment elevation duringtesting, however the patient is symptom free during that time period. Which condition is likely comorbid

A

Diabetes mellitus

How well did you know this?
1
Not at all
2
3
4
5
Perfectly
70
Q

A 4 month infant presents with repeated episodes of tachypnea, poor feeding and failure to thrive.On PE, the child is pink and her extremities are warm. There is aloud, harsh holosystolic murmur that is herd best at the left sternal border. A systolic throll is palpated over the region .On CXR the heart is enlarged and there are increased pulmonary vascular markings. The most likely diagnosis in this patient is.
And what is the underlying anatomic location?

A

Ventricular septal defect

Membranous interventricular septum

How well did you know this?
1
Not at all
2
3
4
5
Perfectly
71
Q

What indicates the presence of an acute myocardial infarction in this patient

A

elevated serum troponins and ST elevation

-not:
t wave inversion, ST depression, Q waves

How well did you know this?
1
Not at all
2
3
4
5
Perfectly
72
Q

A 13 year old female presents to the office, she has no significant past medical history. ON cardiac auscultation you determine that there is fixed splitting of her second heart sound.

A

A defect in atrial septum.

How well did you know this?
1
Not at all
2
3
4
5
Perfectly
73
Q

A previously healthy 12 year old is brought to ER after being stung by a bee on his right forearm. Fifteen minutes later he complained of shortness of breath and was observed by his parents to be wheezing. On exam he is tachycardiac and hypotensive, repsiratory examination reveals wheezing with mild subcostal retractions,. his capillary refill time is delayed. What drug to give

A

Epinephrine: first line for anaphalactic shock

How well did you know this?
1
Not at all
2
3
4
5
Perfectly
74
Q

A Swan-Ganz Catheter is placed to evaluate a postoperative patient with hyptension and suspected heart failure. The calculated cardiac index is low and the pulmonary artery wedge pressure that is obtained is 2 mm Hg (normal 6-15 mm Hg). What is the msot likely diagnosis in this patient?

A

Hemorrhage - really low left atrium pressure.

How well did you know this?
1
Not at all
2
3
4
5
Perfectly
75
Q

67 year old’s ECG shows a wide QRS complex and broad notched R wave in V6. In addition there are absent R and prominant S waves in V1. What auscultory finding is consistent?

A

paradoxically split S2

How well did you know this?
1
Not at all
2
3
4
5
Perfectly
76
Q

2 year old is brought to office because of poor activity tolerance and increased dyspnea with exertion. In the office as the toddler is crying she appears to be cyanotic. A heave is present anteriorly along the left sternal border. A harsh systolic ejection murmur is heard over the upper left sternal border which decreases as the baby is crying. A single S2 is auscultated. What is the underlying condition and likely mechanism for the systolic murmur that is auscultated

A

Tetralogy of Fallot

Subvalvular pulmonic stenosis

How well did you know this?
1
Not at all
2
3
4
5
Perfectly
77
Q

54 year old with hypercholesterolemia is about toe begin on atorvastatin. Which serum chemistry should be obtained as a baseline

A

Serum transaminases

How well did you know this?
1
Not at all
2
3
4
5
Perfectly
78
Q

Which of the following are platelet P2YI ADP receptor

  • enoxaparin
  • bivalirudin
  • aspirin
  • clopidogrel
  • tirofiban
A

Clopidogrel

How well did you know this?
1
Not at all
2
3
4
5
Perfectly
79
Q

Chronically ischemic tissue that shows viability on nuclear imaging and goes from hypokinetic to normal wall motion after revascularization is known as:

A

Hybernating myocardium is myocardium that is hypokinetic due to longstanding severe ischemia, but has viability and wall motion improves after revascularization.

80
Q

What might you find in the physical exam of an ischemic heart
disease patient that would indicate ischemic papillary muscle
dysfunction?
a) Diastolic murmer indicating aortic insufficiency b) Systolic murmer indicating aortic stenosis c) Diastolic murmer indicating mitral stenosis d) Systolic murmer indicating mitral regurgitation

A

Papillary muscle disfunction from ischemia can cause mitral regurgitation which would produce a pansystolic murmer.

81
Q

What indicates a positive or markedly positive stress test

A

Positive test:
• 1mm of horizontal or downsloping ST-segment depression •

Markedly positive:
• Fall in systolic blood pressure, ventricular arrhythmias, >2mm ST segment depression,
ischemic changes within 3 minutes or lasting more than 5 minutes after the end of exercise, or patient exercise<2 minutes

82
Q

With warfarin treatment you can get painful bruises and blistered areas around the calf and heel which progress proximally and affect the lower leg as well. In addition you can get large hemorrhagic bullae. What is the MOA

A

Effects of warfarin on protein C synthesis

83
Q

22 yr old is brought to hospital after work related accident. Physicians suspect internal hemorrhage and the blood pressure is 90/50 mmHG. What is the intracellular compensatory mechanism in the arteriolar smooth muscles in response to her hypotension

A

increase in IP3

84
Q

What is the MOA of ezetimide

A

Decreased gastrointestinal absorption of cholesterol

85
Q

A drug decreases pulse rate, increases both diastolic and systolic blood pressure and signifcantly increases peripheral resistance.

A

Norepinephrine.

86
Q

Stages of heart failure

A, B, C, D

A
Stage A: high risk for HF without structural heart disease or symptoms
 Hypertension
 Atherosclerotic heart disease
 Diabetes
 Obesity
 Metabolic syndrome
 Family history of cardiomyopathy
Stage B: Structural heart disease without the development of HF
 Prior myocardial infarction
 Depressed LV ejection fraction
 Left ventricular hypertrophy
 Asymptomatic valvular heart disease

Stage C: Structural heart disease with current or prior symptoms
(heart failure syndrome)

Stage D Heart Failure End-stage HF
Refractory symptoms requiring special intervention:
 Inotropes
 LVAD
 Transplant
 Hospice
87
Q

What are 5 symptoms of CHF in an infant

A
  1. Tachypnea and diaphoresis during feeds
  2. Easy fatigability
  3.   Irritability
  4.   Decreased volume of feeds
  5. Poor weight gain

•  “Failure to thrive”

88
Q

How can atrial enlargement result in dysphagia and hoarseness.
Describe the layers of the pericardium?
What innervates the pericardium and where can referred pain go?.

A

The most posterior part of the heart is the left atrium enlargement can cause dysphagia (due to
compression of the esophagus) or hoarseness (due to compression of the left recurrent laryngeal
nerve, a branch of the vagus).

Pericardium consists of 3 layers (from outer to inner):
􀂃 Fibrous pericardium
􀂃 Parietal layer of serous pericardium
􀂃 Visceral layer of serous pericardium
Pericardial cavity lies between parietal and visceral layers.

Pericardium innervated by phrenic nerve. Pericarditis can cause referred pain to the shoulder.

89
Q

what are the 5 stages of left ventricle diastole,systole

A
  1. Isovolumetric contraction—period
    between mitral valve closing and aortic
    valve opening; period of highest O2
    consumption
  2. Systolic ejection—period between aortic
    valve opening and closing
  3. Isovolumetric relaxation—period between
    aortic valve closing and mitral valve
    opening
  4. Rapid filling—period just after mitral
    valve opening
  5. Reduced filling—period just before mitral
    valve closing
90
Q
*Jugular venous pulse 
a wave
c wave
x descent 
v wave 
y descent
A

a wave—atrial contraction. Absent in atrial
fibrillation (AF).

c wave—RV contraction (closed tricuspid valve
bulging into atrium).

x descent—downward displacement of closed
tricuspid valve during rapid ventricular
ejection phase. Reduced or absent in tricuspid
regurgitation and right HF because pressure
gradients are reduced.

v wave—increased right atrial pressure due to filling
(“villing”) against closed tricuspid valve.

y descent—RA emptying into RV. Prominent
in constrictive pericarditis, absent in cardiac
tamponade.

91
Q

How do each of these effect murmurs

  1. Inspiration (increase venous return to right atrium)
  2. Hand grip (increase afterload)
  3. Valsalva (phase II), standing up (decrease preload)
  4. Rapid squatting (increase venous return, increase preload, increase afterload)
A
  1. increase intensity of right heart sounds
  2. increase intensity of MR, AR, and VSD murmurs
    decrease hypertrophic cardiomyopathy and AS, mitral valve prolapse later onset of click/murmur
  3. decrease intensity of most murmurs (including AS) due to decreased preload- increase intensity of hypertrophic cardiomyopathy murmur
    MVP: earlier onset of click/murmur
  4. decrease intensity of hypertrophic cardiomyopathy murmur
    increase intensity of AS, MR, and VSD murmurs
    MVP: later onset of click/murmur
92
Q

Diastolic heart murmurs

A

[Diastolic]
-Aortic regurgitation (AR)
High-pitched “blowing” early diastolic decrescendo murmur. Long diastolic murmur, hyperdynamic pulse, and head bobbing when severe and chronic. Wide
pulse pressure. Often due to aortic root dilation, bicuspid aortic valve, endocarditis,
rheumatic fever. Progresses to left HF.

[Mitral stenosis (MS)]
Follows opening snap (OS due to abrupt halt in leaflet motion in diastole, after
rapid opening due to fusion at leaflet tips). Delayed rumbling mid-to-late diastolic
murmur (decreased interval between S2 and OS correlates with increased severity). LA&raquo_space; LV
pressure during diastole. Often a late (and highly specific) sequela of rheumatic
fever. Chronic MS can result in LA dilatation.

Continuous PDA Continuous machine-like murmur. Loudest at S2. Often due to congenital rubella or prematurity. Best heard at left infraclavicular area

93
Q

First degree AV block

Mobitz Type 1 2nd degree

A

First degree AV block
Diagnosis:
 PR Interval > 200 msec with one to one relationship between P waves and QRS complexes
 Mechanism: -Slowing of conduction in the AV node
 Etiology:
-Increased Parasympathetic Tone, Medications that slow AV Nodal conduction, AV nodal Ischemia
-Chronic degenerative disease, Inferior Myocardial infarction
 Clinical Presentation:
-Elderly
- Beta-blockers, Calcium channel blockers, Digitalis
 Treatment:
- Usually no treatment, avoid drugs that will further impair AV conduction

Mobitz Type 1 2nd degree (Wenckebach):
 Diagnosis:
-Intermittent failure of AV conduction with some p waves not followed by QRS complex, constant P to P intervals and prolongation of the PR interval before block
 Mechanism:
-Impaired conduction in the AV Node with intermittent block
 Etiology: Increased Parasympathetic Tone, Medications that slow AV Nodal conduction, AV nodal Ischemia, inflammation
 Clinical Presentation:
- Inferior MI, Lyme myocarditis, congenital AV block , post OHS
- Trained Athlete, Beta-blockers, Calcium channel blockers
 Treatment:
-Usually reversible with removing medications or with time
- May need atropine, isoproterenol or temporary pacing

Mobitz Type 2: Dropped beats that are not preceded by a change in the length of the PR interval (as in type I).
May progress to 3rd-degree block. Often treated with pacemaker

94
Q

What is dilated cardiomyopathy

A
Dilated cardiomyopathy (DCM) is a condition in which the heart's ability to pump blood is decreased because the heart's main pumping chamber, the left ventricle, is enlarged and weakened. In some cases, it prevents the heart from relaxing and filling with blood as it should.
decreases contractility and leads to lower ejection fraction and possibly heart failure
95
Q

46 year old woman is treated with an antiarrhythmic drug. After nine months of therapy, she returns with symptoms of arthralgias and a malar rash and she is diagnosed with a systemic lupus like syndrome. Which of the following drugs is most commonly associated with this side effect.

A

Procainamide

96
Q

What is the presumed mechanism for wide complex monomorphic tachycardia

A

Slow conduction and unidirectional block

97
Q

What is the mechanism of action for lidocaine?

A

It binds to open and inactivated sodium channels.

Primarily inactivated sodium channels which are found in ischemic tissue due to the inability of Na/K ATPase to hyperpolarize the cell.

98
Q
59 year old woman presents to the office concerned with recurrent palpitations accompanied by dizziness. the day prior she experiences a near syncopal episode. A holter monitor is ordered which demonstrates a long QT interval and isolated episodes of polymorphic ventricular tachycardia. Which of the following medications could have precipitated the arrhythmia demonstrated in this patient. 
Metoprolol 
Diltiazem
Nadolol
Sotolol
Lidocaine
A

Sotolol

99
Q

a 43 year old has been told she has a heart murmur but does not know the detais. She reports no increased fatigue, dyspnea or palpitations. On PE the HR is 78/min and BP is 126/68 mm Hg. Lungs are clear to auscultation. A loud S1 is heard, S2 is normal. There is no S3 or S4. There is a low pitched mid diastolic rumbling murmur that is immediately preceded by a sharp, high pitched snap. The murmur does not vary with inspiration and is heard best at apex.

A

Mitral stenosis, likely pathogenesis is immune mediated valve damage.

100
Q
Which of these suggests a mitral stenosis has progressed in severity 
-increased murmur intensity
increased intensity of S1
P2 is accentuated
murmur duration shortens
S2-opening snap interval lengthens
A

P2 in accentuated

101
Q

27 year old woman presents to the ER complaining of palpitations and dizziness. She states that the palpitations came on abruptly while she was seated at her desk and reports no prior episodes. Her pulse is 170 per minute and regular, he PE is otherwise unremarkable. Her thythm strip confirms a rate of 170. THe QRS complexes are narrow and there are no visible P waves, vagal stimulation fails to slow her heart rate. What would be the drug of choice to terminate this arrhythmia

A

Adenosine

AV nodal reentrant tachycardia - it opens AV noe potassium channels causing hyperpolarization.

102
Q

55 year olf with a history of heart murmur is brought to ER after syncopal episode. He states he was running in the park and suddenly lost consciousness. ON PE he has a sustained apical impulse, he has a 3/6 systolic ejection murmur peaking in midsystole, that is loudest at the tright second intercostal space radiating to the neck.

  1. What is the diagnosis and the heart sounds upon inspiration.
  2. What could be a congenital risk factor?
  3. What is causing the clinical manifestation of syncope
A

This is aortic stenosis
A2 is after P2 and upon inspiration its paradoxical because they are closer
Congenital anomaly of the valve - bicuspid

-fixed stenosis prevents the augmentation of cardiac output

103
Q
Which of the following drugs are good for atrial flutter
Disopyramide
Nifedipine
Amiodarone
Amlodipine
Metoprolol
A

Metoprolol

104
Q

65 yr old with PMH significant for type 2 diabetes and hyperlipidemia presents to ER with epigastric discomfort of two hours duration. PE findings are notable for irregular rhythm, presence of an S3 as well as bibasilar rales. An ECG reveals ST elevation as well as Q waves. the patient is taken for PCI and a stent is placed in the left anterior descending coronary artery. her symptoms improve. One hour later the patient dvelops acute dyspnea, on exam there is a holosystolic murmur at the apex which fades quickly, bibasilar rales and wheezing also. What would the primary goal at this moment?

A

Papillary muscle rupture - decrease afterload

105
Q

73 r old with CAD presents to the office concerned with increasing dyspnea on exertion. on PE his heart rate is 85 /min and regular MP is 106/80 mm Hg. His PMI is shifted laterally. S1 and S2 are normal. An S3 is present. there is a holosystolic blowing murmur at the left sternal border that increases with inspiration. His liver is palpable below the costal margin and he has 2+ pedal edema. What is the most likely cause of this patient’s problem?

A

Left ventricular systolic dysfunction.

Left sternal border murmur that increases with inspiration:
Tricuspid valve regurg - increase with inspiration - increases preload, more going through the right sided valves, which fits with right heart failure - can’t empty properly, the heart gets bigger, it stretches the annulus of the valve so it doesn’t close properly. The most common cause of right heart failure is?

106
Q

A 16 yr old reports intermittent episodes in which she feels that her heart is racing and jumping in her chest. The palptations are accompanied by shortness of breath. While she is in the office she reports abrupt onset of palpitations and feels mildly dizzy. A rhythm strip is obtained and demonstrates a narrow QRS tachycardia with a regular rate of 160/min. No P waves are evident. A vagal maneuver terminates the arrhythmia and subseqeunt ECG is normal What is the most likely diagnosis in the patient?

A

Atrioventricular nodal reentrant tachycardia.

107
Q

Torsades de pointes

A

Polymorphic ventricular tachycardia,
characterized by shifting sinusoidal waveforms
on ECG; can progress to ventricular
fibrillation (VF).
Long QT interval predisposes to torsades de pointes.
Caused by drugs, decrease K+, decrease Mg2+, congenital abnormalities.
Treatment includes magnesium sulfate.

Drug-induced long QT (ABCDE):
AntiArrhythmics (class IA, III)
AntiBiotics (eg, macrolides)
Anti“C”ychotics (eg, haloperidol)
AntiDepressants (eg, TCAs)
AntiEmetics (eg, ondansetron)
Torsades de pointes = twisting of the points
108
Q

Congenital long QT syndrome

A

Inherited disorder of myocardial repolarization,
typically due to ion channel defects; increase risk of
sudden cardiac death (SCD) due to torsades de
pointes. Includes:
- Romano-Ward syndrome—autosomal
dominant, pure cardiac phenotype (no
deafness).
- Jervell and Lange-Nielsen syndrome—
autosomal recessive, sensorineural deafness.

109
Q

Brugada syndrome

A

Autosomal dominant disorder most common in Asian males. ECG pattern of 👾pseudo-right bundle branch block and ST elevations in V1-V3. increase risk of 👾ventricular tachyarrhythmias and SCD. PreventSCD with implantable cardioverter-defibrillator (ICD).

EKG - type 1, coved type ST segment elevation, type 2 -
saddle back type ST elevation

110
Q

Atrial Fibrillation

Atrial Flutter

Ventricular Fibrillation

A

Atrial fibrillation: chaotic and erratic baseline with no discrete P waves in between irregularly spaced QRS complexes - loss of “a” wave. Irregularly irregular
heartbeat - think Afib.

Most common risk factors include hypertension and coronary artery disease (CAD). Ag, valvular heart disease, congestive heart failure.
-enlarged atrium R or L.
Can lead to thromboembolic events, particularly stroke.
Treatment includes anticoagulation, rate control, rhythm control, and/or cardioversion.

Atrial Flutter - A rapid succession of identical, back-to-back atrial depolarization waves. The identical appearance accounts for the “sawtooth” appearance of the flutter waves.
Treat like atrial fibrillation. Definitive treatment is catheter ablation.

Ventricular Fibrillation - A completely erratic rhythm with no identifiable waves. Fatal arrhythmia without immediate CPR and defibrillation.

111
Q

ANP

BNP

A

Atrial natriuretic peptide:
Released from atrial myocytes in response to increased blood volume and atrial pressure. Acts via cGMP.
Causes vasodilation and decrease Na+ reabsorption at the renal collecting tubule. Dilates afferent renal
arterioles and constricts efferent arterioles, promoting diuresis and contributing to “aldosterone
escape” mechanism.

B-type (brain) natriuretic peptide
Released from ventricular myocytes in response to increased tension. Similar physiologic action to ANP,
with longer half-life. BNP blood test used for diagnosing HF (very good negative predictive value).
Available in recombinant form (nesiritide) for treatment of HF.

112
Q

Describe autoregulation of the following: How blood flow to an organ remains constant over a wide range of perfusion pressures

Heart
Brain
Kidneys 
Lung
Skeletal Muscle 
Skin
A

Mnemonic: CO2, H+, Adenosine, Lactate, K+ (CHALK)

Heart: Local metabolites (vasodilatory): adenosine, NO, CO2, decreased O2

Brain: Local metabolites (vasodilatory): CO2 (pH)

Kidneys: Myogenic and tubuloglomerular feedback

Lungs: Hypoxia causes vasoconstriction

Skeletal muscle Local metabolites during exercise: lactate, adenosine, K+, H+, CO2
At rest: sympathetic tone

*Note: the pulmonary vasculature is unique in
that hypoxia causes vasoconstriction so that
only well-ventilated areas are perfused. In
other organs, hypoxia causes vasodilation.

113
Q

Which congenital defects are associated with following

  1. Fetal alcohol syndrome
  2. Congenital rubella
  3. Down Syndrome
  4. Diabetic mother
  5. Marfan Syndrome
  6. Prenatal lithium exposure
  7. Turner Syndrome
  8. Williams syndrome
  9. 22q11 syndrome
A
  1. Fetal alcohol syndrome
    - VSD, PDA, ASD, tetralogy of Fallot
  2. Congenital rubella
    - PDA, pulmonary artery stenosis, septal defects
  3. Down Syndrome
    -AV septal defect (endocardial cushion defect),
    VSD, ASD
  4. Diabetic mother
    - transposition of the great vessels
  5. Marfan Syndrome
    -MVP, thoracic aortic aneurysm and dissection,
    aortic regurgitation
  6. Prenatal lithium exposure - Ebstein anomaly
  7. Turner Syndrome - Bicuspid aortic valve, coarctation of aorta
  8. Williams syndrome - Supravalvular aortic stenosis
  9. 22q11 syndrome - Truncus arteriosus, tetralogy of Fallot
114
Q

Essential vs secondary hypertension
Hypertensive urgency
Hypertensive emergency

What does hypertension predispose to, 9

A

90% of hypertension is 1° (essential) and related to increased CO or increased TPR. Remaining 10% mostly 2° to renal/renovascular disease (eg, fibromuscular dysplasia [which has characteristic “string of beads”
appearance of renal artery], atherosclerosis) and 1° hyperaldosteronism.

Hypertensive urgency—severe (≥ 180/≥ 120 mm Hg) hypertension without acute end-organ damage.

Hypertensive emergency—severe hypertension with evidence of acute end-organ damage (eg, encephalopathy, stroke, retinal hemorrhages and exudates, papilledema, MI, HF, aortic dissection,
kidney injury, microangiopathic hemolytic anemia, eclampsia: seizures that occur during a woman’s pregnancy or shortly after giving birth.).

PREDISPOSES TO CAD, LVH, HF, atrial fibrillation; aortic dissection, aortic aneurysm; stroke; chronic kidney disease (hypertensive nephropathy); retinopathy.

115
Q

Takotsubo cardiomyopathy

A

Takotsubo cardiomyopathy: “broken heart syndrome”—ventricular apical ballooning
likely due to increased sympathetic stimulation
(s t re s s f u l sit u at ion s).
_________________________

Post-menopausal women
 Can mimic ACS
 Chest pain, diffuse T-wave inversions, + Troponin
 Echo, cardiac cath – LV gram
 LV function normalizes over days or weeks

116
Q

A 67 YO female undergoes an emergency sigmoid
colon resection for perforated diverticulitis.
During induction of anesthesia, she was transiently hypotensive to 70 mm Hg systolic.
Her BP responded promptly to IV fluids.
The operation was “difficult” and there was 1000cc reported blood loss.

It is 11PM, and the nurse calls YOU
to evaluate the pa>ent, who has
suddenly become hypotensive to
60 mm Hg systolic.

Pt is anxious. No SOB or chest pain is reported
•  Vital signs: T 102°F, HR 120, RR 28
•  HEENT: sclera anicteric, mucous membranes dry
•  Neck: No jugular venous disten>on •  Lungs: CTA, equal BS bilaterally
•  Heart: sinus tach, nl S1, S2, no G/R/M
•  Abdomen: incision is intact, minimally distended
•  Skin: warm, pink and well-perfused
No JVD Lungs CTA
WNL

What is the differential diagnosis?

A

. (has sign of infection, hypotension, maybe she is bleeding. and age of cardiovascular risk (maybe cardiac event)

Before PE 
DD: 
•  Hypovolemic	Shock	
–  Bleeding	
–  Inadequate	fluid	resuscita>o

• Cardiogenic Shock
–  MI, arrythmia, pulmonary embolu

•  Distributive Shock
–  Sepsis
_________________________
Significant PE findings
-febrile
-skin is warm pink and well perfused,
-no JVD
-lungs are clear (suggesting that its not a cardiogenic shock)
-normal S1,S2, no S3/S4 , not a primary problem with the heart
-that leaves us with septic shock or hypovolemic shock

so this must be septic shock.

117
Q

What is Beck’s Triad

A

Clinical features of pericardial tamponade

Hypotension
Muffled HS
JV distension (due to impaired filling)

+
tachycardia
pulsus paradoxus

118
Q
BP low
HR high
Skin cool clammy
PVR is high 
Lungs congested 
Heart sounds: S3, S4 
JVD yes 

what is happening?

A

Cardiogenic shock

119
Q
Hypovolemic 
Cardiogenic 
Obstructive
Distributive (sepsis, anaphylaxis vs CNS injury) 
-shocks
Caused by:
Skin:
PCWP (preload):
CO:
SVR: 
Treatment:
A
Hypovolemic:
Caused by: hemmorhage, dehydration, burns.
Skin - cold and clammy
PCWP- really low
CO - low
SVR - high
Treatment: IV fluid 
Cardiogenic: 
Caused by Acute MI, HF, vavlular dyfunction, arrhythmia
Skin - cold clammy -
Preload can be increased or decreased
CO- REALLY LOW
SVR - high 
Treatment: inotropes, diuretics 
Obstructive: 
Caused by cardiac tamponade, pulmonary embolism, tension pneumothorax, 
Skin -cold, clammy
PCWP can be increased or decreased
CO - REALLY LOW
Treatment: relieve obstruction
Distributive:
Caused by sepsis, anaphylaxis, 
Skin- warm
PCWP - low
CO - high
SVR - REALLY LOW 
Tx- IV fluids, pressors 
caused by CNS injury 
Skin - dry
PCWP - low 
CO- low
SVR - REALLY low 
HR - slow**
Tx- IV fluids, pressors
120
Q

Which drugs induce a SLE like syndrome

Look at them blushing, I SHIPP-EM

A
Sulfa drugs, 
Hydralazine, 
Isoniazid,
Procainamide, 
Phenytoin, 
Etanercept 
Methyl-dopa
121
Q

Dilated Cardiomyopathy

A

Most common cardiomyopathy (90% of cases).
Often idiopathic or familial. Other etiologies
include chronic Alcohol abuse, wet Beriberi, Coxsackie B viral myocarditis, chronic Cocaine use, Chagas disease, Doxorubicin toxicity, hemochromatosis, sarcoidosis, peripartum cardiomyopathy.
Findings: HF, S3, systolic regurgitant murmur, dilated heart on echocardiogram, balloon
appearance of heart on CXR.
Treatment: Na+ restriction, ACE inhibitors, β-blockers, diuretics, digoxin, ICD, heart transplant.
Systolic dysfunction ensues. Eccentric hypertrophy A (sarcomeres added in
series). ABCCCD.

122
Q
SA node - the start 
Atrial myocardium
AV node
Bundle of His
Bundle branches
Purkinje net work 
Ventricular myocardium

place in order of slowest to fastest in regards to conduction velocity and include beats

A

Slowest conduction:
1. AV node - .05, 40-55 bpm

  1. Ventricular myocardium -
    .3 - 1.0, no beats
  2. atrial myocardium - 1.0-1.2, no beats
  3. Bundle of his - 1.2 - 2.0, 25-40 bpm
  4. Bundle branches/Purkinje network - 2.0-4.0, 25-40 bpm.
123
Q

What are symptoms and physical findings of left sided and right sided heart failures

A
Left: 
[symptoms]- 
-Dyspnea	 
-Orthopnes	 
-PND	 
-Fatigue	 
[physical findings]
-Tachycardia/Tachypnea	 
-Rales	 
-Cheyne-Stokes	 
-S3	gallop	(Systolic Dysfunction)	
- S4 gallop	 (diastolic dysfunction) 
\_\_\_\_\_\_\_\_\_\_\_\_\_\_\_\_
Right:
[symptoms] 
-Peripheral	edema	 
-RUQ discomfort	 
-Decreased	appetite

[physical findings]

  • Jugular venous distension
  • Hepatomegaly
  • Peripheral edema
  • Ascites
124
Q

Congenital long QT syndrome

LQT1
LQT2
LQT3

A

LQT1 - decreased outward potassium - AD and AR

LQT2 - decreased outward potassium
AD only

LQT3 (sodium channel) - increased inward sodium
AD only

Inherited disorder of myocardial repolarization, typically due to ion channel defects; increase risk of
sudden cardiac death (SCD) due to torsades de pointes. Includes:
ƒ
1. Romano-Ward syndrome—autosomal dominant, pure cardiac phenotype (no deafness).
ƒ
2Jervell and Lange-Nielsen syndrome—
autosomal recessive, sensorineural deafness.

  1. Autosomal dominant disorder most common in Asian males. ECG pattern of pseudo-right bundle branch block and ST elevations in V1-V3. Increaserisk of ventricular tachyarrhythmias and SCD. Prevent
    SCD with implantable cardioverter-deibrillator (ICD).
125
Q

A 22-year-old presents with exertional dyspnea. Physical examination showed blood pressure 170/90 in both arms and a heart rate of 74 beats/ minute. Cardiac auscultation reveals an S4, which is loudest at the apex. On CXR, the cardiac silhouette was at the upper limits of normal and notching of the ribs was noted. An MRI was performed and the results are illustrated. Which of the following physical findings would be most consistent with the likely diagnosis?

A.  Fixed splitting of S1
B. Radial to femoral pulse delay
C.  Continuous machine-like murmur below left clavicle
D.  Capillary pulsations
E.  A blowing decrescendo diastolic murmur over the LSB

A

B - radial to femoral pulse delay

this is a coarctation of the aorta

  • notching will occur before or after the PDA
  • the differential cyanosis will only occur if coarctation is before the open ductus, if there was no open ductus there would be no flow for infants because no intercostals
126
Q
An 32-year-old presents to the the office for a routine examination.  She is otherwise healthy with no significant past medical history.  Her physical examination is significant for an early systolic ejection click as well as a III/ VI crescendo decrescendo systolic ejection murmur heard best at the right second intercostal space.  The murmur decreases with Valsalva. She has no reported history of rheumatic heart disease. Which of the following additional findings would be consistent with her diagnosis?
A.  Mitral valve prolapse 
B.  Ascending aortic dilation 
C.  Patent ductus arteriosus 
D.  Hypertrophic cardiomyopathy 
E.  Atrial septal defect
A

Likely a bicupsid aorta.

B. Ascending aortic dilatation - bicuspid aorta is associated with higher rates of aortic stenosis, aortic dissection and aortic aneurysm.

127
Q

Marfan’s syndrome

A

Marfan syndrome—[AD]!!

connective tissue disorder affecting skeleton, heart,
and eyes. FBN1 gene mutation on chromosome 15 results in defective fibrillin, a glycoprotein
that forms a sheath around elastin. Findings: tall with long extremities; pectus carinatum (more speciic) or pectus excavatum; hypermobile joints; long, tapering fingers and toes (arachnodactyly); cystic medial necrosis of aorta; bicuspid aorta, aortic incompetence and dissecting aortic
aneurysms; loppy mitral valve. Subluxation of lenses, typically upward and temporally.

128
Q

Which of the following additional physical examination findings would be consistent with aortic dissection
A.  Weak or absent left carotid pulse
B.  Wide pulse pressure
C.  Early diastolic decrescendo murmur D.  Weak or absent left brachial pulse E.  All of the above

A

All of the above.

A.  Weak or absent left carotid pulse

B.  Wide pulse pressure - no hypovolemic shock because you aren’t losing blood volume, but it can rupture. Diastolic drops - 40% will have aortic regurgitation - the head bobbing, pulsation in the capillary beds of finger (quinckes), the huge differential in pressure causes the headbobbing (systolic + low diastolic)

C.  Early diastolic decrescendo murmur D.  Weak or absent left brachial pulse E.  All of the above

129
Q

A 69-year-old man presents to the ER with a 1-hour history of acute severe abdominal pain, back pain and nausea. On arrival, his blood pressure was 77/55 mmHg and pulse rate was 140 beats per minute. He appears pale and diaphoretic. Physical examination disclosed tenderness over the epigastrium and left mid abdomen. His extremities are cool to the touch.
Ultrasound reveals an aneurysm that is leaking.

A.  He has an underlying collagen vascular disorder
B.  Screening for this disorder is cost effective
C.  The celiac and superior mesenteric arteries are likely involved
D.  It was caused by immune-mediated destruction of the blood vessel
E.  It was caused by a congenital defect

A

A. most common cause is degenerative

*B. screening for this disorder is cost effective

C. below the renals usually

D. likely caused by just hypertension and atherosclerosis and inflammation

130
Q

What are congenital causes of aneurysms for those under the age of 65\

4 congenital

MELT

A

Syndromic connective tissue disorders

  • Marfan Syndrome: fibrillin
  • Ehlers-Danlos (vascular): type III pro-collagen
  • Loeys-Dietz Syndrome: TGF-beta receptor
  • Turner syndrome: XO
131
Q

Thoracic aortic aneurysm and abdominal aortic aneurysm - causes and symptoms

what are the symptoms of rupture

A
  • *thoracic is acquired
  • mostly degenerative (hypertension, smoking, atherosclerosis)
  • acquired infection (SYPHILIS, arteritis of the vasovasorum within the media)

Often asymptomatic
-most common presenting symptom is pain - acute will imply and impending rupture or dissection

{Aortic}
-environmental risk is more important as dissection is more likely degenerative
-smoking, COPD, prior aneurysm, CAD, hypertension
-hereditary disorders (Marfan, E-D), genetics
-usually asymptomatic until rupture - symptoms can include back, flank, abdominal or groin pain, occassionally compressive symptoms,
PE- unreliable but you can find a pulsatile abdominal mass in the epigastric region

[rupture]
-abdominal or back pain plus low blood pressure
-ruptured AAA - groin pain, syncope, paralysis and flank mass-
rupture of AAA has 80% mortality
-Physical signs; hypotension, cold clammy extremities, other signs of shock - it takes time to get septic shock but this is very acute

FA Associated with cystic medial degeneration. Risk factors include hypertension, bicuspid aortic
valve, connective tissue disease (eg, Marfan syndrome). Also historically associated with 3°
syphilis (obliterative endarteritis of the vasa vasorum). Aortic root dilatation may lead to aortic
valve regurgitation.

132
Q

What are physical findings of peripheral arterial disease

  • arterial insufficiency
  • listen for bruits (renal, iliac, femoral)
  • palpate pulses (femoral, popliteal, posterior tibial, dorsalis pedis)
A

Look for signs of arterial insufficiency

  • absent or diminished pulses
  • **-atrophy of calf muscles ) bilateral because occlusion in one is leading to lack of exercise entirely
    - disease of distal superficial femoral artery is most common!
  • male erectile dysfunction - (aortoiliac area, pain in thigh and buttocks
  • thickened toe nails
  • (not predictive sign)- loss of hair below the knees)
  • thin, shiny skin (venous disease skin gets thicken, hemosideron deposition)
  • non-healing wounds
  • **dependent rubor - ischemia interferes with normal autonomic autoregulation of arterioles so they no longer constrict in the presence of pressure (turn pale when press skin)
  • dependent because if you lift the leg then it turns white
133
Q

Hypovolemic shock

Causes, skin, PCWP, CO, SVR, treatments

A
[Hypovolemic]
Caused by: hemmorhage, dehydration, burns, trauma 
Skin - cold and clammy
PCWP- really low - low preload
CO - low
SVR - HIGH
Treatment: IV fluid 

**Look for narrow pulse pressure >750 mL of blood loss, systolic BP is normal and becomes reduced
Diastolic BP is increased because you have severe narrowing of blood vessels
-cardiac compliance is normal or low

134
Q

Distributive Shock (3 kinds)

Causes, skin, PCWP, CO, SVR, treatments

A
Distributive:
Caused by sepsis, anaphylaxis, 
Skin- warm
PCWP - low, low filling pressures NO JVD 
CO - high - decreased MAP tho because of volume depletion 
Contractility - low 
SVR - REALLY LOW **warm, dry skin, pink
Tx- IV fluids, pressors 

SEPTIC SHOCK - profound (vasoplegic shock) vasodilation and permeability (disrupted cell-cell contacts) + decreased perfusion in periphery despite increased vasopressors
-initiated by danger signals - pathogen associated molecular patterns, recognized leading to robust inflammatory response

NEUROGENIC SHOCK - sympathetic tone secondary to spinal cord injury

ANAPHYLACTIC SHOCK - histamine, leukotriene C4, prostaglandin D2 release cause profound vasodilation

135
Q

Cardiogenic shock

Causes, skin, PCWP, CO, SVR, treatments

A
Cardiogenic: 
Caused by Acute MI, HF, vavlular dyfunction, arrhythmia
Skin - cold clammy -
Preload can be increased or decreased
*YES JVD 
CO- REALLY LOW
SVR - high -vasoconstriction 
Low MAP - low SV 
Treatment: inotropes, diuretics
  • leading cause of death for patients with acute MI
  • primary pump failure: myocardial infarction (>40% ventricle)
  • cardiomyopathy (fulminant myocarditis, dilated cardiomyopathy)
  • rhythm disturbance, valvular heart disease

pathophysiology

  1. coronary occlusion/myocardial ischemia
  2. profound depression of myocardial contractility
  3. reduced cardiac output
  4. low blood pressure
    * *5. worsening coronary insufficiency (perfusion of coronaries depends on pressure in aortic root so if pressure falls, worse perfusion)

Findings:

  1. Hypotension and low cardiac output - tachycardia, faint pulses
  2. hypoperfusion: agitation, disorientation, or lethargy
    - cool, clammy, cyanotic extremities
    - oliguria (low urine output)
  3. congestion: elevation of jugular venous pressure, pulmonary rales, third heart sounds.
136
Q

Obstructive shock

Causes, skin, PCWP, CO, SVR, treatments

A
Obstructive: 
Caused by cardiac tamponade, pulmonary embolism (increased afterload), tension pneumothorax, 
Skin -cold, clammy
PCWP can be increased or decreased
[*YES JVD]
Filling pressures are HIGH 
PVR is HIGH 
CO - REALLY LOW
Treatment: relieve obstruction
-IMPAIRED venous return to R or l ventricle 

Clinical features of pericardial tamponade (1. hypotension, 2. muffled HS, 3. JV distension - Beck’s triad)

    • *look for PULSUS PARADOXUS: an abnormally large decrease in systolic blood pressure and pulse wave amplitude during inspiration. The normal fall in pressure is less than 10 mmHg.
  1. ELECTRICAL ALTERNANS - alternating QRS morphology. The heart is floating in a sac so the vector is shifting all the time
Mechanism of pulsus paradoxus: with -increased venous return to right side of the heart > there is no change to the volume of the left ventricle, the reason for this is because the right ventricle is complaint,  when there is volume around the pericardial sac, the pressure is on the walls of both ventricles. The right ventricle is not compliant as the excess volume comes into it then, the only complaint area that is not really feeling the pressure is the septum which pushes into the left and impedes the blood filling the LV from the pulmonary circulation,  LV volume declines, drop in LV pressure worsens. 
\_\_\_\_\_\_\_\_\_\_\_\_\_\_\_\_\_\_\_\_\_\_\_\_\_\_\_\_\_\_\_\_\_
Tension pneumothorax: 
Clinical features 
1. Absent breath sounds 
2. Jugular Venous distension 
3. Tracheal deviation 
-CVP is high 
(no muffled heart sounds unlike tamponode)
137
Q

Describe the pathway of smooth muscle contraction and relaxation

both feature increased calcium

A

Contraction:

  1. action potential reaches Smooth MUSCLE CELL > membrane depolarization
  2. L-type voltage gated calcium channel > influx
  3. Increased intracellular calcium
  4. Calcium binds to CALMODULIN
  5. Ca2+-calmodulin activate MYOSIN LIGHT CHAIN KINASE (MLCK)
  6. MLCK phosphorylates light chains in myosin heads and increases myosin ATPase activity
  7. active myosin crossbridges slide along actin and create muscle tension

Relaxation

  1. Agonist binds to receptor on ENDOTHELIAL CELL (acetylcholine, bradykinin)
  2. increased calcium
  3. NO synthase active > (L-arginine to NO)
  4. NO quickly diffuses into the smooth muscle cell
  5. NO converts GTP > cGMP
  6. cGMP activates MYOSIN-LIGHT Chain phosphatase (MLCP) relaxation
138
Q
  1. Within 1 hour after a gunshot wound to the abdomen, a 19-year-old man exhibits tachycardia. His skin is cool and clammy to the touch, and blood pressure is 80/30 mm Hg. Which of the following organ-specific changes is most likely to occur within 2 days after this injury?
    (A) Acute hepatic infarction
    (B) Cerebral basal ganglia hemorrhage (C) Renal passive congestion
    (D) Pulmonary diffuse alveolar damage (E) Gangrenous necrosis of the lower legs
A

Answer 1. (D) The patient is quickly going into shock after trauma. So-called shock lung, with diffuse alveolar damage, is common in this situation. Infarction of the liver is uncommon because of this organ’s dual blood supply. Basal ganglia hemorrhages are more typical of hypertension, not hypotension with shock. Passive congestion is less likely because of the diminished blood volumes and tissue perfusion that occur in shock. Gangrene requires much longer to develop and is not a common complication of shock.

139
Q
  1. A study of atheroma formation leading to atherosclerotic complications evaluates potential risk factors for relevance in a population. Three factors are found to play a significant role in the causation of atherosclerosis: smoking, hypertension, and hypercholesterolemia. These factors are analyzed for their relationship to experimental models for atherogenesis. Which of the following events is the most important direct biologic consequence of these factors? (A) Endothelial injury and its sequelae (B) Conversion of smooth muscle cells to foam cells (C) Alterations of hepatic lipoprotein receptors (D) Inhibition of LDL oxidation (E) Alterations of endogenous factors regulating vasomotor tone
A

Answer 2. (A) Atherosclerosis is thought to result from a form of endothelial injury and the subsequent chronic inflammation and repair of the intima. All risk factors; including smoking, hyperlipidemia, and hypertension, cause biochemical or mechanical injury to the endothelium. Formation of foam cells occurs after the initial endothelial injury. Although lipoprotein receptor alterations can occur in some inherited conditions, these account for only a fraction of cases of atherosclerosis, and other lifestyle conditions do not affect their action. Inhibition of LDL oxidation should diminish atheroma formation. Vasomotor tone does not play a major role in atherogenesis.

140
Q
  1. A cohort study is performed involving healthy adult men and women born 20 years ago. They are followed to assess development of atherosclerotic cardiovascular diseases. Multiple laboratory tests are performed yearly during this study. An elevation in which of the following test results is most likely to indicate the greatest relative risk for development of one of these diseases? (A) Anti-proteinase 3 (PR3) (B) C-reactive protein (CRP) (C) Cryoglobulin (D) Erythrocyte sedimentation rate (ESR) (E) Platelet count
A
  1. (B) CRP is an acute phase reactant that increases in response to inflammation. It causes endothelial cell activation, promotes thrombosis, and increases leukocyte adhesiveness in developing atheromas. Since atherogenesis is partly an inflammatory process, the CRP is an independent predictor of cardiovascular risk. PR3 is one type of antineutrophil cytoplasmic autoantibody (ANCA) associated with some vasculitides such as microscopic polyangiitis. Cryoglobulins may be found with some forms of immune complex mediated vasculitis. The ESR (“sed rate”) is a very non-specific indicator of inflammation and therefore the internists’ least favorite test; the ESR is best known to be markedly elevated with giant cell arteritis. Though platelets play a role in atheroma formation, the actual number of platelets is not a predictor of atherogenesis.
141
Q
  1. A 49-year-old man is feeling well when he visits his physician for a routine health maintenance examination for the first time in 20 years. On physical examination, his vital signs are temperature, 37°C; pulse, 73/min; respirations, 14/min; and blood pressure, 155/95 mm Hg. He has had no serious medical problems and takes no medications. Which of the following is most likely to be the primary factor in this patient’s hypertension? (A) Increased catecholamine secretion (B) Renal retention of excess sodium (C) Gene defects in aldosterone metabolism (D) Renal artery stenosis (E) Increased production of atrial natriuretic factor
A
  1. (B) This patient has essential hypertension (no obvious cause for his moderate hypertension). Renal retention of excess sodium, which is thought to be important in initiating this form of hypertension, leads to increased intravascular fluid volume, increase in cardiac output, and peripheral vasoconstriction. Increased catecholamine secretion (as can occur in pheochromocytoma), gene defects in aldosterone metabolism, and renal artery stenosis all can cause secondary hypertension. Hypertension secondary to all causes is much less common, however, than essential hypertension. Increased production of atrial natriuretic factor reduces sodium retention and reduces blood volume.
142
Q
  1. An 80-year-old man with a lengthy history of smoking survived a small myocardial infarction several years ago. He now reports chest and leg pain during exercise. On physical examination, his vital signs are temperature, 36.9°C; pulse, 81/min; respirations, 15/min; and blood pressure, 165/100 mm Hg. Peripheral pulses are poor in the lower extremities. There is a 7-cm pulsating mass in the midline of the lower abdomen. Laboratory studies include two fasting serum glucose measurements of 170 mg/dL and 200 mg/dL. Which of the following vascular lesions is most likely to be present in this patient? (A) Aortic dissection (B) Arteriovenous fistula (C) Atherosclerotic aneurysm (D) Glomus tumor (E) Polyarteritis nodosa (F) Takayasu arteritis (G) Thromboangiitis obliterans
A

C) Abdominal aneurysms are most often related to underlying atherosclerosis. This patient has multiple risk factors for atherosclerosis, including diabetes mellitus, hypertension, and smoking. When the aneurysm reaches this size, there is a significant risk of rupture. An aortic dissection is typically a sudden, life-threatening event with dissection of blood out of the aortic lumen, typically into the chest, without a pulsatile mass. The risk factors for atherosclerosis and hypertension underlie aortic dissection. An arteriovenous fistula can produce an audible bruit on auscultation. Glomus rumors are usually small peripheral masses. Polyarteritis nodosa can produce small microaneurysms in small arteries. Takayasu arteritis typically involves the aortic arch and branches in children. Thromboangiitis obliterans (Buerger disease) is a rare condition with occlusion of the muscular arteries of the lower extremities in smokers.

143
Q
  1. A 19-year-old man has had a low-grade fever for 3 weeks. On physical examination, his temperature is 38.3°C, pulse is 104/min, respirations are 28/min, and blood pressure is 95/60 mm Hg. A tender spleen tip is palpable. There are splinter hemorrhages under the fingernails and tender hemorrhagic nodules on the palms and soles. A heart murmur is heard on auscultation. Which of the following infectious agents is most likely to be cultured from this patient’s blood? (A) Viridans streptococci (B) Trypanosoma cruzi (C) Coxsackievirus B (D) Candida albicans (E) Mycobacterium tuberculosis (F) Pseudomonas aeruginosa
A

.6. (A) Prolonged fever, heart murmur, mild splenomegaly, and splinter hemorrhages suggest a diagnosis of infective endocarditis. The valvular vegetations with infective endocarditis are friable and can break off and embolize. The time course of weeks suggests a “subacute” form of bacterial endocarditis resulting from infection with a less virulent organism, such as Streptococcus viridans. Trypanosoma cruzi and coxsackievirus B are causes of myocarditis. Candida is not a common cause of infective endocarditis, but may be seen in immunocompromised patients. Tuberculosis involving the heart most often manifests as pericarditis. Pseudomonas is more likely to cause an acute form of bacterial endocarditis that worsens over days, not weeks; this organism is more common as a nosocomial infection or in injection drug users.

144
Q
  1. A 50-year-old man has sudden onset of severe substernal chest pain that radiates to the neck. On physical examination, he is afebrile, but has tachycardia, hyperventilation, and hypotension. No cardiac murmurs are heard on auscultation. Emergent coronary angiography shows a thrombotic occlusion of the left circumflex artery and areas of 50% to 70% narrowing in the proximal circumflex and anterior descending arteries. Which of the following complications of this disease is most likely to occur within 1 hour of these events? (A) Ventricular fibrillation (B) Pericarditis (C) Myocardial rupture (D) Ventricular aneurysm (E) Thromboembolism
A

Answer 7. (A) In the period immediately after coronary thrombosis, arrhythmias are the most important complication and can lead to sudden cardiac death. It is believed that, even before ischemic injury manifests in the heart, there is greatly increased electrical irritability. Pericarditis and rupture occur several days later. An aneurysm is a late complication of healing of a large transmural infarction; a mural thrombus may fill an aneurysm and become a source of emboli. If portions of the coronary thrombus break off and embolize, they enter smaller arterial branches in the distribution already affected by ischemia.

145
Q
  1. During the past year, a 34-year-old woman has had palpitation, fatigue, and worsening chest pain. On physical examination, she is afebrile. Her pulse is 75/min. respirations are 15/min, and blood pressure is 110/70 mm Hg. Auscultation of the chest indicates a midsystolic click with late systolic murmur. A review of systems indicates that the patient has one or two anxiety attacks per month. An echocardiogram is most likely to show which of the following? (A) Aortic valvular vegetations (B) Pulmonic stenosis (C) Mitral valve prolapse (D) Patent ductus arteriosus (E) Tricuspid valve regurgitation
A

Answer 8. (C) These findings indicate a floppy mitral valve, a condition that usually is asymptomatic. When symptomatic, it can cause fatigue, chest pain, and arrhythmias. Pulmonic stenosis is most often a congenital heart disease. Valvular vegetations suggest endocarditis, and a murmur is likely to be heard with infective endocarditis causing valvular insufficiency. A patent ductus arteriosus causes a shrill systolic murmur. Tricuspid regurgitation is accompanied by a rumbling systolic murmur.

146
Q

A four-year-old girl who is below the 5th percentile for height and weight for age becomes easily fatigued. On physical examination, she appears cyanotic. Her temperature is 37°C, pulse is 82/min, respirations are 16/min, and blood pressure is 105/65 mm Hg. Arterial blood gas measurement shows decreased oxygen saturation. One month later, she has fever and obtundation. A cerebral CT scan shows a right parietal, ring-enhancing, 3-cm lesion. Which of the following congenital heart diseases is the most likely diagnosis? (A) Atrial septal defect (B) Bicuspid aortic valve (C) Coarctation of the aorta (D) Patent ductus arteriosus (E) Truncus arteriosus (F) Ventricular septal defect

A

.Answer 9. (E) Cyanosis at this early age suggests a right-to-left shunt; truncus arteriosus, transposition of the great arteries, and tetralogy of Fallot are the most common causes of cyanotic congenital heart diseases. The cerebral lesion suggests an abscess as a consequence of septic embolization from infective endocarditis, which can complicate congenital heart disease. Atrial septal defect, patent ductus arteriosus, and ventricular septal defect initially lead to left-to- right shunts. Coarctation is not accompanied by a shunt and cyanosis. In most cases, a bicuspid valve is asymptomatic until adulthood, and there is no shunt.

147
Q

Evolution of myocardial infarction
4 stages

Commonly occluded coronary arteries: LAD > RCA > circumlex. Symptoms: diaphoresis, nausea, vomiting, severe retrosternal pain, pain in left arm and/or jaw, shortness of breath, fatigue.

A

0-1 day: early coagulative necrosis, release of necrotic cell contents into blood, edema, hemorrhage, wavy fibers. Neutrophils appear, reperfusion injury associated with generation of free radicals, leads to hypercontraction of myofibrils through increase of free calcium influx
COmplications: ventricular arrhythmia, cardiogenic shock

1-3 days: extensive coagulative necrosis, tissue surrounding infarct shows acute inflammation with neutrophils.
Complications: postinfarction fibrinous pericarditis

3-14 days - macrophages, then granulation tissue at margins. Free wall rupture >tamponade;
papillary muscle rupture
> mitral regurgitation;
interventricular septal rupture
due to macrophage-mediated
structural degradation. LV pseudoaneurysm (risk of
rupture).

2 weeks to several months 
-contracted scar complete 
-Dressler syndrome, HF,
arrhythmias, true ventricular
aneurysm (risk of mural
thrombus).
148
Q
  1. A 50-year-old man with a lengthy history of diabetes mellitus and hypertension has had pain in the left shoulder and arm for the past 12 hours. He attributes the pain to arthritis and takes acetaminophen. Over the next 6 hours, he develops shortness of breath, which persists for 2 days. On day 3, he visits the physician. On physical examination, his temperature is 37.1°C, pulse is 82/min, respirations are 18/min, and blood pressure is 160/100 mm Hg. Laboratory studies show that the total creatine kinase (CK) activity is within reference range, but the troponin I level is elevated. The patient is admitted to the hospital and continues to experience dyspnea for the next 3 days. On day 7 after the onset of shoulder pain, he has a cardiac arrest. Resuscitative measures are unsuccessful. Postmortem examination shows a large transmural infarction of the left anterior free wall with rupture and hemopericardium. Which of the following statements is best supported by these clinical and autopsy data? (A) Infarction did not develop until day 5 or day 6 after the episode of chest pain. (B) The normal CK level obtained on day 3 excludes the possibility of infarction within the preceding 72 hours. (C) He had an acute infarction occurring on the day he developed shoulder pain. (D) A CK-MB fraction determination would have detected acute infarction on day 3. (E) A second acute infarction on day 6 or day 7 caused myocardial rupture within several hours
A

Answer 10. (C) The kinetics of creatine kinase (CK), CK-MB, and troponin I elevations after a myocardial infarction (MI) are important. 6-12 hrs after an MI, peaks at about 16 to 24 hours, and return to normal within 48 hours. Troponin I levels begin to increase at 4 hours but remain elevated for 7 to 10 days. Total CK activity is a sensitive marker for myocardial injury in the first 24 to 48 hours. CK-MB offers more specificity, but not more sensitivity. Myocardial rupture occurs 5 to 7 days after myocardial necrosis. This patient had an MI on the day of the shoulder pain. When he saw the physician on day 3, the CK levels had returned to normal, but troponin I levels remained elevated. Three days later, the infarct ruptured

149
Q

Diagnosis of myocardial infarction

A

In the first 6 hours, ECG is the gold standard.
Cardiac troponin I rises after 4 hours (peaks
at 24 hr) and is increase for 7–10 days; more speciic
than other protein markers.
CK-MB rises after 6–12 hours (peaks at
16–24 hr) and is predominantly found
in myocardium but can also be released from skeletal muscle. Useful in diagnosing
reinfarction following acute MI because levels return to normal after 48 hours.
Large MIs lead to greater elevations in troponin
I and CK-MB. Exact curves vary with testing procedure.
ECG changes can include ST elevation
(STEMI, transmural infarct), ST depression
(NSTEMI, subendocardial infarct),
hyperacute (peaked) T waves, T-wave
inversion, new left bundle branch block, and
pathologic Q waves or poor R wave progression
(evolving or old transmural infarct).

150
Q

What is the most common cause of thoracic aneurysm?

WHat is the major complication

A

Classically seen in
tertiary syphilis

which causes endarteritis of the vasa vasorum in the adventitia resulting in luminal narrowing, decreased flow and atrophy of the vessel wall. Results in a “TREE BARK” appearance of aorta

The major complication is dilation of the aortic valve root resulting in aortic valve insufficiency.

oTher complications include compression of the mediastinal structures like the airway or esophagus and thombosis + emboli

151
Q

What is high yield to know about the abdominal aortic aneurysm ?

Location:

Patient population

Presents

Complications

A

BELOW the renal arteries, ABOVE the aortic bifurcation

Primarily due to atherosclerosis, classically seen in male smokers >60 yrs resulting in atrophy and weakness of the wall.

Pulsatile abdominal mass that grows with time

Major complication is rupture, specially when >5 cm in diameter, presents with triad of hypotension, pulsatil mass and flank pain.

Other complications include compression of local structures (ureter and thrombosis/embolism

152
Q

What are the two small vessel diseases caused by hypertension

A
  1. Hyaline arteriolosclerosis with benign HT or diabetes
  2. Hyperplastic arteriolosclerosis with malignant HT

Hyaline arteriolosclerosis is caused by proteins leaking into the vessel wall producign vascular thickening. Results in reduced vessel caliber with end organ ischemia, classically produces glomerular scarring (arteriolonephrosclerosis that slowly progresses to chronic renal failure
Benign HTN is mild or moderate, clinicall silent vessels and organs are damaged slowly over time.

Hyperplastic arteriolosclerosis involves thickening of vessel wall by hyperplasia of the smooth muscle (onion skin appearance). consequence of malignant hypertension. Results in reduced vessel caliber with end organ ischemia. May lead to fibrinoid necrosis of the vessel wall with hemorrhage. classically causes acute renal failure with characteristic flea bitten appearance.
Malignant HTN is severe elevation in bp >180/120 - presents with acute end organ damage (acute renal failure, headache and papilledema and is a medical emergency.

153
Q

What are the new 2017 guidelines for high blood pressure

A

Normal: Less than 120/80 mm Hg;

Elevated: Systolic between 120-129 AND diastolic less than 80;

Stage 1: Systolic between 130-139 OR diastolic between 80-89;

Stage 2: Systolic at least 140 or diastolic at least 90 mm Hg;

Hypertensive crisis: Systolic over 180 and/or diastolic over 120, with patients needing prompt changes in medication if there are no other indications of problems, or immediate hospitalization if there are signs of organ damage.

154
Q

dressler syndrome

A
Dressler syndrome (autoimmune-mediated post-MI fibrinous pericarditis, 2 weeks to several months after
acute episode)
155
Q

Kaposi’s sarcoma

A

Endothelial malignancy most commonly of the skin, but also mouth, GI tract, and respiratory tract.
Associated with HHV-8 and HIV. Rarely mistaken for bacillary angiomatosis, but has lymphocytic
infiltrate.

Presents as purple patches plaques and nodules on the skin. May also involve visceral organs. Classically seen in older eastern european males, AIDs, and transplant recipients.

156
Q

Angiosarcoma

A

malignant proliferation of the endothelail cells, highly aggressive. Common sites include skin breast and liver. Liver angiosarcoma is associated with exposure to polyvinyl cholride, arsenic and thorotrast.

FA:
Rare blood vessel malignancy typically occurring in the head, neck, and breast areas. Usually in
elderly, on sun-exposed areas. Associated with radiation therapy and chronic postmastectomy
lymphedema. Hepatic angiosarcoma associated with vinyl chloride and arsenic exposures. Very
aggressive and difficult to resect due to delay in diagnosis.

157
Q

A 60 year-old male with a history of stage IIB rectal cancer was brought to the ED by his son for 2 weeks of progressive shortness of breath. He had an indwelling central venous catheter with a portal reservoir for chemotherapy. On physical examination, the patient was ill-appearing. His initial vital signs were as follows: temperature 97.2°F, blood pressure 84/53 mmHg, respiratory rate 38 breaths per minute. An indwelling catheter was present in the right chest wall with the portal reservoir removed. Pulmonary examination demonstrated rales bilaterally. Cardiac auscultation was significant for a 3/6 high-pitched pan-systolic murmur that was best heard at the left sternal border. The murmur is augmented with inspiration. There is a large vegetation on the tricuspid valve. There are pulmonary nodules.

Which of the following statements is true with regard to the clinical and pathologic features of his underlying disorder?
A. The most likely pathogen is Strep viridans
B. Only native valves are affected by this disease
C. The disorder is associated with narrowing of the valve
D. The pulmonary nodules are formed from immune complex deposition
E. Stroke symptoms can occur in these patients

A

The pulmonary nodules are due to septic emboli breaking off from the tricuspid.

Most likely pathogen is S. aureus
Prosthetic valves are better
It doesn’t cause stenosis, it causes regurg
pulmonary nodules are form the platelets

Answer: stroke symptoms can occur in these patients

158
Q

Bacterial endocarditis

A
Fever (most common symptom), new murmur
Roth spots (round white spots on retina surrounded by hemorrhage  A ), Osler nodes (tender raised lesions on inger or toe pads  
due to immune complex deposition), 
-Janeway lesions (small, painless, erythematous lesions on palm or sole)  glomerulonephritis, septic arterial or pulmonary emboli, splinter hemorrhages  on nail bed. Multiple blood cultures necessary for diagnosis.
ƒ
Acute—S aureus (high virulence).
Large vegetations on previously normal
valves. Rapid onset.
Subacute—viridans streptococci (low
virulence). Smaller vegetations on
congenitally abnormal or diseased valves.
Sequela of dental procedures. Gradual
onset.
S bovis (gallolyticus) is present in colon cancer,
S epidermidis on prosthetic valves. Endocarditis may also be nonbacterial
(marantic/thrombotic) 2° to malignancy, hypercoagulable state, or lupus.

Mitral valve is most frequently involved. Tr i cuspid valve endocarditis is associated with IV drug abuse (don’t “tri” drugs). A ssociated
with S aureus, Pseudomonas, and Candida.
Culture ⊝; most likely Coxiella burnetii,
Bartonella spp., HACEK (Haemophilus,
Aggregatibacter (formerly Actinobacillus),
Cardiobacterium, Eikenella, Kingella) ♥ Bacteria FROM JANE ♥:
Fever Roth spots Osler nodes Murmur Janeway lesions Anemia Nail-bed hemorrhage Emboli

159
Q
A	27-year-old	male	presented	to	the	Emergency	Room	with	 weakness	accompanied	by	progressive	dyspnea	on	exertion.		He	 reports	no	past	medical	history	but	notes	that	he	has	been	 experiencing	“flu-like”	symptoms	for	the	past	10	days,	described	as	 non-productive	cough,	fatigue	and	myalgia.		On	physical	 examination,	he	was	febrile.		His	heart	rate	was	120	beats	per	 minute	and	his	BP	was	90/60	mm	Hg.		His	rhythm	was	irregularly	 irregular. An	S1	and	S2	were	noted In	addition,	a	systolic	murmur	 loudest	at	the	apex,	and	having	a decrescendo	quality	was	 auscultated. Bibasilar	pulmonary	rales were	also	noted.		The	patient	 was	stabilized	and	a	myocardial	biopsy	performed	which is demonstrated.	
Reveals lymphocyte rich myocarditis 
What	is	the	likely	diagnosis	in	 this	patient?
A. Acute	myocardial	infarction 
B. Viral	myocarditis 
C. Rheumatic	fever 
D. Lyme	Disease 
E. Chagas Disease
A

Viral myocarditis - most common cause

160
Q

An 11 year old male presents to the ER with fever, joint pain and swelling, along with shortness of breath. The symptoms have been present now for 4 days. Two days ago, his right knee was painful and swollen, but today it has improved. The joints involved today include the right ankle and left knee. They are quite tender, painful and also swollen. The shortness of breath occurs with walking. Exam: VS T 38.2, P 160, RR 32, BP 100/60. He is tired appearing with tachypnea and tachycardia. Heart sounds are tachycardic with a holosystolic murmur 3/6 heard at apex with radiation to axilla. No gallops are heard. His left knee is swollen and extremely tender with warmth. A diagnostic workup is begun and a myocardial biopsy is obtained.

Which of the following statements is true with regard to the clinical and pathologic features of his underlying disorder?
A. Chronic disease results in valvular incompetency
B. Bacterial pathogens invade and damage the heart
C. Damage at this stage can affect all layers of the heart
D. The pulmonary valve is commonly affected
E. This is a common (>30%) sequellae of untreated infection

A

This is acute rheumatic fever

Damage at this stage can affect all layers of the heart.

161
Q

A previously healthy 32-year-old man presented to the ER with pain in the left side of the chest of 4 hours duration. The pain was sharp, radiated to the left trapezius ridge, was worse on inspiration and relieved somewhat by sitting up and leaning forward. His pulse was 100 bpm and regular, BP was 130/80 without a paradoxical pulse and his temperature was 37.9 degrees. A three-component friction rub was heard along the LSB. An ECG revealed diffuse ST segment elevation. He had no cardiac risk factors and no past medical conditions but had a “flu-like” illness in the prior month. He was treated with ibuprofen with improvement in symptoms. He returned two weeks later with chest discomfort and dizziness. Physical exam revealed muffled heart sounds, BP of 96/60 with a pulsus paradoxus of 25 mm Hg.

An echocardiogram revealed pericardial effusion with early tamponade. He underwent pericardiocentesis with drainage of 900 ml of blood stained fluid. His symptoms improved and he was discharged on ibuprofen and colchicine.
Which of the following was likely to be present at catheterization before the pericardiocentesis?
a. Prominent V wave on RA tracing
b. Absent Y descent on RA tracing
c. Prominent X and Y descents on RA tracing
d. Square root sign on RV tracing
e. Inspiratory increase in JVP

A

Description is of Acute viral pericarditis - or idiopathic

Heart is constantly being compresssed, Y descent is when fluid leaves into the ventricles but you can only have more filling if there is space.
B. absent y descent.

  • Prominent V wave indicates tricuspid regurg.
  • Prominant X and Y descents are indicative of restrictive pericarditis
  • square root sign is also show on pericardial constriction
  • inspiratory increase in JVP - they should decrease because you increase more filling of the heart, if JVP increases, that is mostly seen in pericardial constriction.
162
Q

What kind of murmur does an ASD cause?

A

1 A midsystolic pulmonary flow or ejection murmur, resulting from the increased blood flow across the pulmonic valve, is found in 90% of a large cohort of ASD patients. Up-to-date says that “the characteristic cardiac findings of an ASD include a midsystolic pulmonary flow or ejection murmur accompanied by a fixed split second heart sound”

Midsystolic murmur.

“I was doing some Kaplan questions and realized there is a discrepancy between first aid and our lectures slides/lilly. For ASD, our slides say it causes a systolic murmur on the upper left sternal boarder due to increased flow across the pulmonic valves. However, in first aid it says there is a middiastolic murmur at the left lower sternal boarder due to increased flow across the tricuspid valve.

I read Lilly and it mentions it is possible to have both murmurs. However, in regard to the exam and STEP, which murmur should we more so associate with it?”

A: Probably the most important thing to know about ASDs is that
Patients with ASDs can have a murmur
The murmur is NOT caused by the flow of blood through the ASD- why? The low velocity shunt flow across the ASD (due to the low pressure differential between the left and right atria) produces insufficient turbulence for a murmur to be heard

So what causes the murmur that you can hear? It is the increased volume of blood presented to the right side of the heart that causes a FLOW murmur across the right-side valves-especially the pulmonic valve during systole. You would therefore expect:

The above is what I would consider most important- that being said; you CAN also hear a tricuspid flow murmur in diastole if you listen VERY carefully😊 as described below!

A middiastolic murmur of low to medium frequency due to high flow across the tricuspid valve may also be heard with careful auscultation- (meaning it is not easy to hear) and patients need to have a larger left-to-right shunt for that to be present

163
Q

Acute pericarditis

A

Inlammation of the pericardium. Commonly presents with sharp pain (can refer to shoulder), aggravated
by inspiration, and relieved by sitting up and leaning forward.
Often complicated by pericardial
effusion.

Presents with:
-pericardial friction rub “sandpaper” (with 3 components, ventricular systole, diastolic filling and atrial contraction).
-ECG changes include
WIDEspread ST-segment elevation and/or PR depression.
-Fever

Causes include idiopathic (most common; presumed viral), conirmed infection (eg,
Coxsackievirus, adenovirus, echovirus), neoplasia, autoimmune (eg, SLE, rheumatoid arthritis), uremia, cardiovascular (acute STEMI or Dressler syndrome), radiation therapy.

164
Q

Rheumatic fever

A

A consequence of pharyngeal infection with
group A β-hemolytic streptococci. Late
sequelae include rheumatic heart disease,
which affects heart valves—mitral > aortic&raquo_space;
tricuspid (high-pressure valves affected most).
Early lesion is mitral valve regurgitation;
late lesion is mitral stenosis. Associated
with Aschoff bodies (granuloma with giant
cells [blue arrows in A ]), Anitschkow cells
(enlarged macrophages with ovoid, wavy,
rod-like nucleus [red arrow in A ]), q anti-
streptolysin O (ASO) titers.
Immune mediated (type II hypersensitivity);
not a direct effect of bacteria. Antibodies
to M protein cross-react with self antigens
(molecular mimicry).
Treatment/prophylaxis: penicillin.

165
Q

Cardiac tampanode

beck is using a pillow

A

Compression of the heart by luid (eg, blood, effusions [arrows in A ] in pericardial space) pr CO. Equilibration of diastolic pressures in all 4 chambers. Findings: Beck triad (hypotension, distended neck veins, distant heart sounds), q HR, pulsus
paradoxus. ECG shows low-voltage QRS and electrical alternans (due to “swinging” movement of
heart in large effusion). Pulsus paradoxus—r in amplitude of systolic BP by > 10 mmHg during inspiration. Seen in
cardiac tamponade, asthma, obstructive sleep apnea, pericarditis, croup.

166
Q

Syphilitic heart disease

A

3° syphilis disrupts the vasa vasorum of the
aorta with consequent atrophy of vessel wall
and dilatation of aorta and valve ring. May see calciication of aortic root, ascending aortic arch, and thoracic aorta. Leads to “tree
bark” appearance of aorta.
Can result in aneurysm of ascending aorta or aortic arch, aortic insuficiency.

Classically the cause of thoracic aorta aneurysm leading to dilation of the aortic root.

167
Q

Streptococcus bovis

A

Streptococcus bovis
Gram ⊕ cocci, colonizes the gut. S gallolyticus
(S bovis biotype 1) can cause bacteremia and
*subacute endocarditis and is associated with
colon cancer.
Bovis in the blood = cancer in the colon.

168
Q

3-4 months of achine pain in left calf, pain experienced during walking and relieved with rest. He reports he wakes at night with pain in his foot that is releived by dagling his leg. He has a 30 pack year smoking history but quit one year ago. Physical examination reveals absent left dorsalis pedis and posterior tibial pulses Which of the following additional symptoms or signs would be consistent?

Thickened skin with hemosiderin deposits
Numbness and tingling in the left lower leg
Ankle brachial index of 1.2
Perisistent low grade fever and fatigue
Dependent rubor in left leg

Commonly disease in what would explain these symptoms

A

Peripheral arterial disease

A: dependent rubor in left leg

Left superficial femoral artery

169
Q

Guy had an MI 2 hours ago. Light microscopy would reveal what?

A

-wavy myofibrils and contraction bands at infarct edges

170
Q

Patient with a PMH of hodgkin disease receiving radiation presents to the office complaining of 6 months of progressive dyspnea. Patient has gained 25 lbs during preceding 2 months. Noted increased abdominal girth as well as marked lower extremity swelling. Before her present complaints, she had been in good health and was taking no medications. Physical examination reveals pitting lower extremity edema, hepatomegaly and tense ascites. Breath sounds are diminished at the bases bilaterally. CV examination is remarkable for a resting tachycardia and jugular venous distention that worsens with inspiration. What is the likely diagnosis
What auscultory finding would correlate

A

Constrictive pericarditis

Early diastolic knock on cardiac auscultation

171
Q

35 yr old with history of IVDA presents with a 10 day history of intermittent fever, chills and increased fatigue. On auscultation he has 2/6 holosystolic murmur that is loudest at the lower left sternal border and that worsens with inspiration. Which finding would be expected in an xray

A

Multiple cavitating pulmonary nodules on chest x ray

Tricuspid vegetation from endocarditis. `

172
Q

Biopsy reveals aschoff bodies, 3 weeks after having an upper respiratory infection, she is unable to participate in gym activity due to increased exertional dyspnea.
What murmur corresponds
In addition to the heart defects, what would be common to this disorder

A

Holosystolic murmur at the apex radiating to the axilla.

Migratory polyarthritis

173
Q

21 year old with no medical problems and takes no medications. ON PE, BP is 128/73 and pulse rate is 56/min. There is a grade 2/6 early systolic murmur along the left sternal border that worsens with Valsalva. An S4 is present. He does experience dizziness when he exercises. Echocardiogram shows marked septal hypertrophy with an ejection fraction of 65%, marked atrial enlargement and reduced early diastolic filling. What pathologic finding would be on microscope
What medication would be good for him to take?

A

Disarray of the cardiomyocytes with fibrosis

Metoprolol

174
Q
Which of these would occur with cardiac tampanode 
-loss of circulating blood volume
impaired left  ventricular filling
increased systemic vascular resistance 
impaired right ventricular filling 
decreased myocardial contractability
A

-impaired left ventricular filling

175
Q
32 year old firefighter, presents with dyspnea on exertion. He reports that he has trouble keeping up. He reports a nocturnal cough, a 10 poud weight loss and fatigue. PE reveals elevated JVP and decreased breath sounds at the bases bialterally. Leftward displacement of the PMI and an S3. He has a holosystolic murmur at the apex radiating to the axilla. Echo reveals left ventricular systolic dysfunction and dialted ventricles. Exposure to which of the following would explain his diagnosis?
-radiation
-tuberculosis
-amyloid L chains
-alcohol
Group A beta hemolytic strep.

What is causing his systolic murmur?

A

Alcohol

Murmur results from dilation of the valve annulus

176
Q

You are evaluating a sixteen year old. Blood pressure in the right arm is 180/95. Palpating the brachial and femoral pulses you find the femoral pulse is diminished in amplitude and delayed compared to the brachial pulse. What sound would you hear?

As you continue the eval, cardiac auscultation reveals an early systolic ejection click followed by a brief systolic murmur. What is also likely diagnosable in this patient

A

Continuous murmur over the midback

Bicuspid aortic valve

177
Q

What is the direct mechanism by which nitroglycerin alleviates anti-ischemic effect

A

increasing venous capacitance

178
Q

73 yr old man with PMH of poorly controlled hypertension presents with severe substernal chest pain described as sharp and tearing feeling. He states pain was 10/10 in intensity at onset and radiates to the back. on PE his BP was 170/100 mmHG in the left arm. What is the likely inciting event?

A

Intimal tear in the ascending aorta

179
Q
25 yr old with AVRT/ Which of the following should be avoided? 
Ibutilide
procainamide
propafenon
verapamil
A

verapamil

180
Q

*Why is ASD closure contraindicated with Eisenmenger

A

1 most people with ASDs DO NOT get eisenmenger’s (10%)- it is more common in VSD (50%) and PDA (almost all these patients will get Eisenmengers if the PDA is not ID and closed).

In the Congenital Heart Defects lecture slide below, it says that ASD closure is contraindicated once Eisenmenger’s is present – could you elaborate on why that is?

So there are two thoughts to why this is so, and I will tell you that up-to-date says it is a “complex decision” so these patients should be referred to specialists. Clearly– “contraindicated” may not be a 100% hard and fast rule. That being said, let’s explore the possibilities.

With ASD, the larger volume of blood entering in to the pulmonary circulation results in pulmonary arteriole remodeling and can induce pulmonary hypertension which is- for all intensive purposes- identical to idiopathic pulmonary hypertension- In the RV, concentric hypertrophy develops as the RV works against the pressure load and as the pulmonary HTN worsens, the load on the ventricle will worsen and the ventricle will dilate. Symptoms of RH failure can occur. In addition, the worsening pulmonary HTN results in a continued increase in RV pressure causing a R to L shunt to occur (central cyanosis and clubbing can occur).

When R to L shunt occurs- it indicates that the pulmonary HTN is severe and likely to be irreversible. At this point then, it is associated with significantly reduced life expectancy of the patients, regardless of the mode of therapy. In other words, the thinking is that surgery will not reverse the damage at this point and really offers no benefit to the patient. Moreover, there is a school of thought that maintaining the intraatrial communication provides a mechanism to maintain the cardiac output at the expense of desaturation, and there may be some benefit to this.

181
Q

How do each of the following local regulators affect peripheral resistance

NO
endothelin
H+
Oxygen
Adenosine 
Prostaglandins
A
NO - decrease 
endothelin - increases
H+ - decreases
Oxygen - increases
Adenosine - decreases 
Prostaglandins - decreases
182
Q

Hypoplastic left heart syndrome

A
  • small left atrium and ventricle
  • ascending aorta is hypoplastic
  • aortic and mitral valves are atretic

Two requirements

  1. PDA to ensure adequate systemic perfusion from RV to aorta
  2. Nonrestrictive atrial septal defect to ensure adequate mixing of oxygenatd and deoxygenated blood
183
Q

Do patients with hypertrophic cardiomyopathy normally have a smaller LV cavity size? Or is this only found in patients with restrictive cardiomyopathy? Additionally, does atrial enlargement only occur with restrictive cardiomyopathy and not with hypertrophic?

A

There is asymmetric hypertrophy of the left ventricular wall in HOCM, typically affecting the septum- the wall of the ventricle is stiff and this impairs filling- there can also be a component of dynamic obstruction to flow caused by the anterial MV leaflet- so you can have LVH and can get left atrial enlargement as well. With restrictive CM- the diagnosis does not require you to have thickened walls- both ventricles are affected by the infiltrative process, both ventricles will have trouble filling, and in this case- BOTH atria will be dilated leading to “owl eye” appearance on echocardiography

184
Q

With regard to Aortic Dissection and the presence of HYPERTENSION or HYPOTENSION;

A

Dissections are associated with hypertension as an initial physical finding- because essential HTN is a risk factor for AD to begin with - in addition, dissection causes pain which leads to increased catecholamines. Hypotension is an ominous finding early on- indicating a catastrophic dissection- dissection of the coronaries, pericardium or a leak.

185
Q

What is dependent rubor

A

The reason that dependent rubor occurs is that there is autonomic dysfunction caused by the severe ischemia- so that when the leg is dependent- the dermal arterioles do not constrict in response to the increased hydrostatic pressure. When a leg is elevated (any leg) above the heart- you should have enough blood flow to the extremity to be able to overcome the effect of gravity- but with severe ischemia- the stenosis creates such low pressure that the blood can’t be pumped against gravity and the extremity perfusion will be significantly impaired. In patients with lighter skin tone, the extremity will appear white, with darker skin tone, the capillary refill will be poor.

186
Q

What is AL amyloidosis

A
  1. AL amyloid
    – Most common type – Igg light chain produced by clonal population of
    plasma cells in bone marrow – Multi-organ infiltration – Up to 50% have cardiac involvement – 5% have cardiac involvement alone – Age > 50, male 2:1
187
Q

Which of the following changes would be expected with vagal stimulation of the
pacemaker cells in the SA node?
A. A more negative maximum diastolic potential
B. A more negative threshold potential
C. A decrease in the slope of phase 4
D. A more positive maximum diastolic potential
E. A and B are correct
F. A and C are correct

A

A & C are correct .

188
Q

IVC compression

A

Inferior vena cava (IVC) compression is common in women during the third trimester of pregnancy.
The large
uterus compresses the IVC, decreasing venous
return to the heart. This reduction in preload
reduces stroke volume, thus reducing cardiac
output. Recall that mean arterial pressure =
cardiac output × total peripheral resistance;
an acute decrease in either of these parameters
will reduce blood pressure. Pregnant women
can avoid this problem by placing a pillow under
their right side or by lying on their left side
to remove the weight of the gravid uterus from
the IVC.

189
Q

Takayasu arteritis

A

This girl has
Takayasu arteritis, a large-vessel vasculitis usually
found in women <40 years of age of Asian
descent. It is characterized by a thickening of
the aortic arch and/or the proximal great vessels.
The most prominent feature is weak
pulses in the upper extremities. Complications
of untreated late-stage Takayasu include aortic
aneurysms and typically involve the aortic
arch. This leads to narrowing, or possible obliteration,
of the major arteries associated with
the aortic arch. Treatment involves high doses
of oral prednisone that are tapered over many
months as the clinician and patient agree to
minimize the adverse effects of corticosteroids.

190
Q

Mitral regurg after MI, is likely caused by occlusion of what?

A

Posterior descending artery
This patient most
likely suffered from acute mitral regurgitation
secondary to rupture of the posterior papillary
muscle. The anterior and posterior papillary
muscles anchor the chordae tendineae, which
prevent the cusps of the mitral valve from being
forced into the left atrium. An occlusion
of the posterior descending artery can lead to
an infarction of the posterior papillary muscle
and subsequent rupture of the muscle several
days later. Patients will present with a sudden
onset of pulmonary edema and frequently cardiogenic
shock. The anterior papillary
muscle is supplied by both the left anterior descending
artery and the left circumflex artery.
Because of its dual blood supply, the anterior
papillary muscle is less likely to rupture after
a MI. Furthermore, the patient’s initial MI involved
the posterior aspect of the myocardium

191
Q

An aortic aneurysm can lead to ischemia of what part of the GI?

A

Sigmoid colon

The inferior mesenteric
artery (IMA) originates from the aorta inferior
to the renal arteries and superior to the
bifurcation of the aorta into the common iliac
arteries. The IMA supplies blood to the distal
one-third of the transverse colon, descending
and sigmoid colons, and the upper portion of
the rectum. The IMA may sometimes be sacrificed
during an infrarenal aortic aneurysm repair
rather than being reattached to a healthy
segment of aorta. Usually there is enough collateral
flow to the hindgut from the superior
mesenteric artery and the hypogastric arteries
that the loss of the IMA does not result in costep

192
Q

ruptured aortic aneurysm of aortic root
-skin has several nodular lesions present throughout his trunk and extremities

What is the etiology?

A

Disruption of the vasovasorum

Syphilitic aortitis is
characterized by obliterative endarteritis of the
vasa vasorum of the tunica media. This disruption
of the vasa vasorum can lead to an aneurysm,
typically involving the ascending aorta,
and is a manifestation of the tertiary stage of
the disease. Syphilitic aneurysms are often
complicated by atherosclerosis. The patient’s
skin lesions are likely “gummas” of tertiary
syphilis, which appear as fibrous nodules on
the skin surface. Less commonly, these lesions
can be ulcerative.

193
Q

Kawasaki disease

A

Kawasaki disease, a vasculitis
of unknown etiology that is hypothesized
to be an infectious or autoimmune response
(ie, molecular mimicry). The major symptoms
include a
1-high fever of more than five days’
duration,
2-bilateral conjunctivitis,
3-lip fissures,
4-strawberry tongue,
5-palmar and plantar desquamation,
6-cervical lymphadenopathy. Minor
criteria may include elevated acute phase reactants,
leukocytosis, thrombocytosis, and mild
elevation of liver function test results. The
most important complication of Kawasaki disease is the development of coronary artery aneurysms,
which can result in MI. IV immunoglobulin
and aspirin is the appropriate therapy.

194
Q

Which barroreceptor responds to low bp

A

-decreased barroreceptor firing in the carotid sinus,
The carotid sinus
baroreceptor sends an afferent signal via the
glossopharyngeal nerve to the medulla, which
in turn responds by increasing sympathetic
outflow. This results in systemic vasoconstriction,
increased heart rate, increased contractility,
and increased blood pressure.

The baroreceptor located
in the aortic arch responds only to an increase
in blood pressure.

195
Q

G proteins, q, i, s

A

Gq - activates phospholipase C, increasing IP3 and calcium, DAG
alpha1, M1, M3

Gi - inhibition of adenylate cyclase (decrease cAMP) - alpha2, M2, D2

Gs - stimulation of adenylate cyclase (increase cAMP) - beta1, beta2, D1